crim pro outline

135
Criminal Procedure Outline McDowell Fall 2010 The Fourth Amendment Exclusionary Rule Does not apply to grand jury proceedings. What is a Search? Katz v. U.S. (1967) Petitioner was convicted under a federal wire fraud statute for submitting wagering information over the phone in a public telephone booth and gov’t sought to introduce his side of the conversation that they had tapped by a device they had stuck to the outside of the booth. Court of appeals held that the recordings were not obtained in violation of the Fourth Amendment because “there was no physical entrance into the area occupied by the petitioner.” The Court ruled that Katz was entitled to Fourth Amendment protection for his conversations and that a physical intrusion into the area he occupied was unnecessary to bring the Amendment into play. The Fourth Amendment protects people, not places. Test: - The concurring opinion introduced a two fold test that: a person must have exhibited an actual (subjective) expectation of privacy and, second, that the expectation be one that society is prepared to recognize as “reasonable.” This is the test used today. - Reasonable expectation of privacy (or legitimate) o If rep, then search and if they are government actors, implicates the 4 th Amendment o If no rep, then no search and there is no need to justify the search - Searches conduced outside the judicial process, without prior approval by judge of magistrate are per se unreasonable under the Fourth Amendment---subject only to a few exception 1

Upload: jessyka109

Post on 25-Nov-2014

131 views

Category:

Documents


0 download

TRANSCRIPT

Page 1: Crim Pro Outline

Criminal Procedure OutlineMcDowell Fall 2010

The Fourth Amendment

Exclusionary RuleDoes not apply to grand jury proceedings.

What is a Search?

Katz v. U.S. (1967)Petitioner was convicted under a federal wire fraud statute for submitting wagering information over the phone in a public telephone booth and gov’t sought to introduce his side of the conversation that they had tapped by a device they had stuck to the outside of the booth. Court of appeals held that the recordings were not obtained in violation of the Fourth Amendment because “there was no physical entrance into the area occupied by the petitioner.” The Court ruled that Katz was entitled to Fourth Amendment protection for his conversations and that a physical intrusion into the area he occupied was unnecessary to bring the Amendment into play. The Fourth Amendment protects people, not places.

Test:- The concurring opinion introduced a two fold test that: a person must have exhibited an actual

(subjective) expectation of privacy and, second, that the expectation be one that society is prepared to recognize as “reasonable.” This is the test used today.

- Reasonable expectation of privacy (or legitimate)o If rep, then search and if they are government actors, implicates the 4th Amendment

o If no rep, then no search and there is no need to justify the search

- Searches conduced outside the judicial process, without prior approval by judge of magistrate are per se unreasonable under the Fourth Amendment---subject only to a few exception

Factors:SUPREME COURT HAS NEVER CLEARLY DEFINED “REASONABLENESS” AS RELATES TO ”REASONABLE EXPECTATION OF PRIVACY”, BUT DECISION AFFECTED BY SEVERAL FACTORS INCLUDING, BUT NOT LIMITED TO:

ASSUMPTION OF RISK; LOCATION; SOCIAL CUSTOM; PAST PRACTICES AND EXPECTATIONS; VANTAGE POINT; LEGALITY AND INTIMACY OF ACTIVITIES; REDUCED EXPECTATION OF PRIVACY; PROPERTY INTERESTS

United States v. White (1971)The issue is whether the Fourth Amendment bars from evidence the testimony of governmental agents who related certain conversations which had occurred between defendant White and a government informant, Harvey Jackson, and which the agents overheard by monitoring the frequency of a radio transmitter carried by

1

Page 2: Crim Pro Outline

Jackson and concealed on his person. The testimony of government agents, relating conversations between a defendant and an uncover informant, overheard via electronic surveillance, is admissible, despite the informant’s unavailability at trial. (expands Katz)

Precident CasesKatz (see above)

Hoffa v. United States (1966)However strongly a defendant may trust a colleague, his expectations in this respect are not protected by the Fourth Amendment when it turns out that the colleague is a government agent regularly communication with authorities. No interest legitimately protected by the Fourth Amendment is involved for that Amendment affords no protection to a wrongdoers misplaced belief that a person to whom he voluntarily confides his wrongdoing will not reveal it.

Lewis v. United States (1966)No warrant to search and seize is required when the Government sends to a defendant’s home a secret agent who conceals his identity and makes a purchase of narcotics from the accused.

Lopez v. United States (1966)No warrant to search and seize is required when an agent carries electronic equipment to record the defendant’s words and the evidence is gathered is later offered in evidence.

- In Katz, the Supreme Court held that the electronic surveillance of Katz’s conversations with others constituted a search and therefore was regulated by the Fourth Amendment. In White, however, the Court held that the Government’s use of an informant “wired” with a transmitter fell outside the scope of the Fourth Amendment.

Smith v. Maryland (1979)Whether the installation and use of a pen register constitutes a search within the meaning of the Fourth Amendment. After a robbery victim had been receiving threatening calls from the robber, she learned his name and a pen register was installed at his house to determine whether he was calling the victim. He was. They held that a pen register was significantly different than acquiring the content of the conversation so that there was no legitimate expectation of privacy because people have no lep in the numbers they dial on their phones. Even if he did have some subjective expectation of privacy, and that the numbers he dialed would remain private, this expectation is not one that society is prepared to recognize as reasonable. The court concludes that in all probability, the petitioner had no actual expectation of privacy in the phone numbers he dialed, and that, even if he did, his expectation was not legitimate. No search.

TEST FROM KATZ

1. Subjective expectation of privacy

2. Expectation of privacy deemed reasonable by society

IF REP, THEN SEARCH AND IF ALSO GOVT ACTION, THEN 4TH APPLIES

2

Page 3: Crim Pro Outline

USE OF BEEPERS

United States v. Knotts (1983)Federal prosecutors put a beeper in a drum of chloroform that was bought by the defendant presumably for the manufacture of drugs. The police used the beeper to follow him around public roads to a private cabin. The court held that “a person traveling in an automobile on public thoroughfares has no reasonable expectation of privacy in his movements from one place to another.” Further the court held that “there is no indication that the beeper was used in any way to reveal information as to the movement of the drum within the cabin, or in any way that would not have been visible to the naked eye from outside the cabin.

United States v. Karo (1984)The police put a beeper in a can of ether and monitored suspect Karo’s movement of the beeper over public roads but also monitor the tracking device in Karo’s home, as well as in two other houses. The Court held that the warrantless monitoring of a beeper in a private residence, a location open to visual surveillance, violates the Fourth Amendment rights of those who have a justifiable interest in the privacy of the residence. In this case, the beeper told them where the ether was in the house and, unlike in Knotts, this could not have been visually verified by one outside the home.

DOG SNIFFS—p. 113

U.S. v. Place (1983)Federal authorities subjected a passenger’s luggage to a sniff test by a dog trained to identify drugs. The court held it was not a search because it did not require opening the luggage, or expose contraband that would otherwise remain hidden from public view. Further, the information obtained was limited.

Illinois v. Caballes (2005)Official conduct that does not compromise any legitimate interest in privacy is not a search subject to the Fourth Amendment. Governmental conduct that only reveals the possession of contraband compromises no legitimate privacy interest. This is because the expectation that certain facts will not come to the attention of the authorities is not the same as an interest in privacy that society is prepared to consider reasonable. The use of a well trained narcotics detection dog one that does not expose noncontraband items that otherwise would remain hidden from public view during a lawful traffic stop, generally does not implicate legitimate privacy interests. In this case, any intrusion on respondent’s privacy expectation does not rise to the level of a constitutionally cognizable infringement. Dissent talks about the dog’s fallibility

Important language in Caballes that is omitted from excerpt here---sniff did not unnecessarily extend the duration of the traffic stop

OTHER FEDERAL CASES ON DOG SNIFFS

U.S. v. Colyer, 878 F2d 469 (D.C. Circuit)

3

Page 4: Crim Pro Outline

Dog sniff of sleeper car on train not a search

U.S. v. Brock, 417 F3d 692 (7th Circuit 2005)

Use of drug detection dog in common area of house to sniff outside resident’s locked bedroom door not a search. Court emphasized that sniff detects only presence of contraband and yields no info about lawful activity

STATE CASES

Myers v. Indiana, 839 N.E.2d 1154 (2007) cert denied

Use of drug detection dog to sniff exterior of students’ cars in public high school parking lot---no search

TYPE OF ACTIVITY CAN BE IMPORTANT IN REP ANALYSIS

More likely to be REP if activities legal rather than illegal

More likely to be REP if activities personal rather than commercial

LOCATION AN IMPORTANT FACTOR IN REP ANALYSIS

Home----greatest protection

Land---Depends

Hester v. United States (1924)The open fields doctrine provides that the entry of an open field does not implicate the Fourth Amendment.

Oliver v. United States (1984)This remains good after Katz. An open field may include any unoccupied or undeveloped area outside of the curtilage of a home. An open field need neither by open nor a field as these terms are used in common speech. An individual may not legitimately demand privacy for activities conducted out of door in fields, except in the area immediately surrounding the home. The curtilage is the area to which extends the intimate activities associate with the sanctity of a man’s home and the privacies of life.

Factors:Factors to Determine whether Curtilage v. Open Field

1. The proximity of the area claimed to be curtilage to the home2. Whether the area is included within an enclosure surrounding the home3. The nature of the uses to which the area is put, and4. The steps taken by the resident to protect the area from observation by people passing by.

** No trespassing signs strengthen a claim but are not a be all, end all.

California v. Ciraolo (1986)

4

Page 5: Crim Pro Outline

It does not follow from the fact that an area is identified as a curtilage that police surveillance of it inevitably constitutes a search. The court concluded that the surveillance of the curtilage of Ciraolo’s home was not a search. Defendant had met the burden of subjective intent by erecting a high fence. When they flew over his house to see the pot plants his expectation of privacy was unreasonable because it was public navigable airspace and anyone could have seen the plants.

Florida v. Riley (1989)A police officer observed the interior of a partially covered greenhouse in Riley’s backyard, while circling 400 feet above the greenhouse in a police helicopter. Helicopter was within legally navigable airspace for that type of craft so that is okay.

1SUPPRESSION MOTIONS

Purpose—To gain the exclusion from trial of evidence that has been discovered, seized or elicited in violation of either the 4th or 5th Amendment. The granting of a Motion to Suppress can result in charges being dismissed or a plea agreement for the Defendant more lenient than what (s)he would have otherwise received.

Mechanics and Content—If there is questionable evidence, the best practice is to file the Motion to Suppress prior to trial. Many jurisdictions, including FL, require that the Motion be filed prior to trial but grant relief from this requirement when it is shown that the Defendant lacked a reasonable opportunity to raise the objection. Jurisdictions vary as to the requirements regarding the content of the Motion to Suppress. In FL, the Motion must clearly state (i) the particular evidence to be suppressed, (ii) the reasons the evidence should be suppressed and (iii) a general statement of the facts on which the Motion is based. Best practice is to make the motion as brief as possible and draft it in a manner that the defense’s theory is not disclosed.

Hearing and Burdens Therein—Defendant (“D”) has the right to a hearing. This hearing is conducted without a jury (or outside the presence of the jury) and, since the rules of evidence do not apply, hearsay may be admitted. D has the burden of going forward and establishing that the 4th or 5th Amendment applies. (i.e.–with 4th—a search and govt. action)and that D has standing. The following apply to burden of persuasion:

If warrant—D has burden of persuasionIf no warrant–State has burden of proving exception to warrant requirementIf fruit of poisonous tree—D must go forward with specific evidence showing taint— State has ultimate burden to show evidence untainted or exceptionIf State claims consent----Burden on State to show consentVoluntariness of confession or complying with Miranda—State has burdenIdentification not tainted----State has burden

4th Amendment issues—Burden is preponderance of evidence5th Amendment issues—preponderance but in some states, clear and convincingIdentification not tainted—clear and convincing

If Motion to Suppress statement as violation of 5th is denied, Defense can still argue to jury that they should disregard it. And, in some states (but not FL) can have 2nd consideration as to admissibility by jury.

Defendant may testify without waiving right to decline to testify at trial. If D testifies at suppression hearing, cross-exam cannot delve into other issues. None of the testimony the Defendant offers at suppression hearing

5

Page 6: Crim Pro Outline

can be introduced at trial but IT CAN BE USED TO IMPEACH DEFENDANT if he takes stand and says something contrary to what said at suppression hearing.

Effect of Ruling—If Motion granted, exclusion of evidence at trial. State can file anInterlocutory Appeal.If Motion denied—The Motion to Suppress must be made again by D when evidence introduced at trial in order to preserve D’s right to appeal the court’s decision.

PROBLEM

Cheryl Anson lives on approximately 14 acres of rural property. The

front portion of the rectangular-shaped property is pasture land. Her house

is behind the pasture land in the center of a 2 acre lot. The area around the

house on the lot is largely covered by a forest of 50-foot high trees that form

a blanket of green leaves that cover the lot and the house. The entire 2 acre

lot is enclosed by a five foot high wooden fence topped with 2 strands of

barbed wire. Acting on a tip, two officers chartered a helicopter to scout

the area. The helicopter hovered about 15 feet above the trees, tilting and

circling six times to gain a line of sight through the trees. When a line of

sight was finally obtained, the officers saw what they recognized as marijuana

plants. Based on these observations, they got a search warrant and seized

the plants. Did the officers’ observations from the helicopter constitute a

search??

WHAT IF OFFICERS CLIMB A LADDER OR FENCE TO VIEW PLANTS WITHIN ONE’S CURTILAGE??

WHAT IF APARTMENT BUILDING NEXT DOOR AND OFFICERS VIEWED FROM 3RD FLOOR OF APARTMENT BUILDING??

California v. Greenwood (1988)Officers searched through plastic garbage bags left on the curb in front of a house for trash pickup. The Court ruled that a person does not have a reasonable expectation of privacy in garbage left outside the curtilage of a home for trash removal. The objective prong of Katz is not satisfied in such circumstances.

6

Page 7: Crim Pro Outline

PROBLEM

WHAT IF YOUR GARBAGE CANS ARE IN YOUR DRIVEWAY 20 FEET FROM THE PUBLIC SIDEWALK AND 50 FEET FROM YOUR HOUSE AND THAT IS WHERE THE TRASH COLLECTION SERVICE COLLECTS YOUR TRASH EACH WEEK-

DEFENSE ARGUMENT??

GOVT ARGUMENT?? Dow Chemical v. U.S. (1986)Agents of EPA photographed Dow’s 2000-acre outdoor industrial complex from different altitudes with a ridiculous camera. The Court, observing that it was important that the area photographed was not an area immediately adjacent to a private home, where privacy expectations are most heightened, upheld the use of the camera to take pictures because the camera remained limited to the outline of the buildings. The fact that human vision is enhanced somewhat does not give rise to constitutional concern.

Kyllo v. United States (2001)Whether the use of a thermal-imaging device aimed at a private home from a public street to detect relative amounts of heat within the home constitutes a “search” within the meaning of the Fourth Amendment. They saw that heat bills were high and thought they were growing pot in the house. They used a thermal imaging cam to detect heat in a growing room. The court held that where, as here, the Government uses a device that is not in general public use, to explore the details of the home that would previously have been unknowable without physical intrusion, the surveillance is a search and is presumptively unreasonable without a warrant.

Bondi v. United States (2000)Squeezing luggage to search for narcotics constitutes a search because there is a physical invasion. However, it is not objectively reasonable to expect privacy if any member of the public could have used his senses to detect everything that the officers observed.

VANTAGE POINT OF GOVT ACTOR CAN MATTER

IF AREA OPEN TO PUBLIC OBSERVATION, NO REP

BUT–IF POSITION OF LEO OR USE OF ENHANCEMENT DEVICE ALLOWS LEO TO SEE OR REVEALS WHAT OTHERWISE ISNOT EXPOSED TO PUBLIC VIEW—GOOD ARGUMENT FOR REP

DRESSING ROOMS AND RESTROOMS

REP????

Completely enclosed dressing room–no door louvers

Same dressing room–sign on mirror–“this area subject to store surveillance.”

Dressing room with door—2 foot gap at top and bottom7

Page 8: Crim Pro Outline

Dressing room with door louvers and common hallway outside

Store security guard watching shopper through peephole in ceiling

Police officer watching man in restroom stall through vents in attic.

Police officer’s observations of man in restroom stall making motions with his feet.

IF DETERMINE 4TH AMENDMENT APPLIES (SEARCH & GOVT ACTION), NEXT QUESTION----

DOES THE DEFENDANT HAVE STANDING TO OBJECT TO THE ADMISSION OF THE EVIDENCE?

STANDING V. IS THERE A SEARCH

DIFFERENTIATION SOMETIMES DIFFICULT1SEARCH----DID GOVT INTRUDE ON ANYONE’S REP??

D HAVE STANDING---DID GOVT INTRUDE ON THIS PARTICULAR DEFENDANT’S REP????

Alderman & Jones pp. 446-447

PERSON CLAIMING VIOLATION MUST BE PERSON WHOSE RIGHTS VIOLATED.

FOR DEFENDANT TO HAVE STANDING TO OBJECT TO ADMISSION OF EVIDENCE,THE DEFENDANT MUST HAVE A LEGITIMATE EXPECTATION OF PRIVACY IN PLACE SEARCHED OR POSSESSORY INTEREST IN ITEMS SEIZED.

The court ruled that a person has standing to contest electronic surveillance and thus is entitled to suppression of conversations, if: (a) government agents unlawfully overheard that person’s conversations regardless of where they occurred, or (b) if the conversations occurred on that person’s premises, whether or not she was present or participated in the conversations.

United States v. Payner (1980)Took things from his briefcase. Can he contest based on Alderman (pg. 448)

Rakas v. Illinois (1978)The prosecutor denied the petitioner-passenger’s standing to object to the lawfulness of the search of the car because neither the car, the shells, nor the rifle belonged to the passengers. The court held that there is no legitimate expectation of privacy interest in being in someone else’s car as a passenger partly because you do not have dominion and control over the car and further, you cannot exclude others from it.

Test:- Whether a disputed search of seizure has infringed an interest of the defendant which the Fourth

Amendment was designed to protect and;- Whether the challenged search or seizure violated that Fourth Amendment right.

8

Page 9: Crim Pro Outline

Simmons v. United States (1968)In order to assert standing when a passenger in a car, an individual must assert ownership over the article in question (such as, in this case, a briefcase) to show that the car was searched without probable cause. This Court held that when a defendant testifies in support of a motion to suppress evidence on Fourth Amendment grounds, his testimony may not thereafter be admitted against him at trial on the issue of guilt unless he makes no objection.

Minnesota v. Olson (1990)Olson was staying as an overnight guest at a friend’s house. The court held that he had standing to contest the warrantless search because of social custom and being “vulnerable while asleep.” They stated there is a legitimate expectation of privacy to shelter your things and yourself in a host’s home.

Minnesota v. Carter (1998)While at a friend’s apartment bagging cocaine, defendant is caught. While trying to assert a legitimate expectation of privacy under Olson, court held that a temporary out of state visitor, present only to bag cocaine (business) does not have a legitimate expectation of privacy. Property used for commercial purposes is treated different for Fourth Amendment purposes than residential property. An expectation of privacy in commercial premises is different from, and less than, a similar expectation in an individual’s home. In some circumstances a worker can claim Fourth Amendment protection under his own workplace. Citing the purely commercial nature of the transaction, the relatively short period of time on the premises, and the lack of any previous connection between the respondents and the householder, the respondent’s situation is closer to that of one simply permitted to be on the premises and that there was no Fourth Amendment violation.

HOW DO YOU SUPPOSE THESE JUSTICES WOULD COME OUT IF FOLLOWING WERE THE SITUATION

You work at the same law firm with Mr. Carter. Mr. Carter comes to your apartment one evening so that the two of you can prepare for a trial the next day. After working for four hours, you decide to share a “joint.” Police officer sees through window, immediately gets warrant and 30 minutes later is in your apartment , seizes marijuana and charges you both with possession.

Rawlings v. Kentucky (1980)The Court ruled that the test enunciated in Rakas---whether the petitioner had a reasonable expectation of privacy in the area searched—is the exclusive test for determining whether a defendant may successfully challenge a search. This case involved drugs that one individual put in another’s purse. When the purse owner emptied the contents---to which several people had access---the owner of the drugs admitted they were his but tried to argue that he had a legitimate expectation of privacy in her purse. The court rejected that notion and further, his contention that he claimed ownership of the drugs so he should be entitled to challenge the search regardless of his expectation of privacy was also rejected.

Probable CauseProbable cause is the traditional standard of the Fourth Amendment. Probable cause to arrest where the facts and circumstances within the officer’s knowledge and of which they have reasonably trustworthy information are sufficient in themselves to warrant a man of reasonable caution in the belief that an offense has been or is being committed by the person to be arrested. The police may either set out the facts and apply for a warrant under a magistrate or an exception to the rule.

9

Page 10: Crim Pro Outline

Spinelli v. United States (1969)After a confidential informant gave information that petition was running a gambling operation the magistrate issued a warrant based primarily on that because the other facts were shady. Probable cause must be determined by a neutral and detached magistrate not by the officer engaged in the often competitive enterprise of ferretting out crime. CI failed under the Aguilar test. the Court went further by requiring that a magistrate must be informed of the "underlying circumstances from which the informant had concluded" that a crime had been committed.

Aguilar v. Texas (1964)Although an affidavit supporting a search warrant may be based on hearsay information and need not reflect the direct personal observations of the affiant, the magistrate must be informed of some of the underlying circumstances relied on by the person providing the information and some of the underlying circumstances from which the affiant concluded that the informant, whose identity was not disclosed, was creditable or his information reliable.

Test:

The two "prongs" of the test are that, when law enforcement seeks a search warrant and a magistrate signs a warrant:

1. The magistrate must be informed of the reasons to support the conclusion that such an informant is reliable and credible.

2. The magistrate must be informed of some of the underlying circumstances relied on by the person providing the information

Draper v. United States (1959)Facts given by the CI in this case were so specific down to the ounce of drugs, the suit the man would be wearing, and what time his train would be arriving, that a magistrate could infer when confronted with such detail that the informant had gained his information in a reliable way.

SO—FROM 1969 UNTIL GATES CASE DECIDED IN 1983, AT LEAST THE WAY MOST LOWER COURTS INTERPRETED AGUILAR-SPINELLI, TEST APPLIED IN DETERMINING WHETHER SUFFICIENT PROBABLE CAUSE WHEN INFORMANT TIP INVOLVED WAS THE TWO-PRONGED A/S TEST---HAD TO ESTABLISH :

1. CREDIBILITY & RELIABILITY (VERACITY) OF INFORMANT.

2. THAT INFORMANT HAD SUFFICIENT BASIS OF KNOWLEDGE

10

Page 11: Crim Pro Outline

Illinois v. Gates (1983)Anonymous letter that a lot of detail about a couple selling drugs (i.e. where they bought and sold from, where they lived, amount of drugs they had). Police officer learned that they had plane tickets just like letter said they would. The police officers obtained a warrant and found a ton of drugs. Police found no violation under the new totality of the circumstances test that replaced Spinelli.

GATES SUMMARY

ABANDONS RIGID APPLICATION OF A-S PRONGS

VERACITY (CREDIBILITY AND RELIABILITY) AND BASIS OF KNOWLEDGE ARE FACTORS TO CONSIDER BUT WEAK PRONGCAN BE OVERCOME BY STRENGTH OF ANOTHERCORROBORATION OF DETAILS OF TIP BY INDEPENDENT POLICE WORK IS IMPORTANT

NEW TEST: TOTALITY OF CIRCUMSTANCES

WHETHER GIVEN ALL CIRCUMSTANCES SET FORTH IN AFFIDAVIT INCLUDING “VERACITY” (CREDIBILITY AND RELIABILITY) AND “BASIS OF KNOWLEDGE”(RELIABILITY OF INFORMATION) OF PERSONS SUPPLYING HEARSAY INFORMATION, THERE IS A FAIR PROBABILITY THAT CONTRABAND OR EVIDENCE OF A CRIME WILL BE FOUND IN A PARTICULAR PLACE

DUTY OF REVIEWING COURT

ENSURE MAGISTRATE HAD SUBSTANTIAL BASIS FOR CONCLUDING THAT PROBABLE CAUSE EXISTED. (Great deference given to magistrate’sdecision)

GATES Totality of Circumstances APPROACH APPLIES TO ALL PC INQUIRIES, NOT JUST THOSE INVOLVING INFORMANTS’ TIPS.

WHAT IF A POLICE OFFICER LIES IN THE AFFIDAVIT FOR PROBABLE CAUSE OR KNOWS THAT WHAT AN INFORMANT IS SAYING IS FALSE---DOES THE DEFENDANT HAVE ANY REMEDY??

Franks v. Delaware (1978)Where the defendant makes a substantial preliminary showing that a false statement knowing and intentionally, or with reckless disregard for the truth, was included by the affiant in the warrant affidavit, the Fourth Amendment requires that a hearing be held at the defendant’s request. In the event that at the hearing the allegation of perjury or reckless disregard is established by preponderance of the evidence, and with the affidavit’s false material set to one side, the affidavit’s remaining content is insufficient to establish probable cause, the search warrant must be voided and the fruits of the search excluded to the same extent as it probable cause was lacking on the fact of the affidavit.

11

Page 12: Crim Pro Outline

1THOUGH FOR SEARCHES, WARRANTS ARE PREFERRED, SEARCHES AND SEIZURES (& DEFINITELY ARRESTS) ARE MORE OFTEN WITHOUT A WARRANT (EXCEPTION TO REQUIREMENT OR REASONABLENESS BALANCING)

WHY ARE WARRANTS PREFERRED??

A SEARCH WARRANT MAY BE SOUGHT FOR:1. Evidence of a crime2. Fruits of a crime 3. Property used in or intended to be used in a crime4. Contraband5. A person to be arrested or who is unlawfully restrained

QUESTIONS TO ASK WHEN SEARCH OCCURS PURSUANT TO WARRANT

1. WAS WARRANT APPLICATION SUFFICIENT?

PROCESS

Officer prepares APPLICATION FOR WARRANT

With APPLICATION-----SWORN AFFIDAVIT(usually from officer)

Affidavit itself must set forth information establishing PROBABLE CAUSE

Can be done over the phone

Often get assistance with above from prosecutor

IF MAGISTRATE SATISFIED THAT PROBABLE CAUSE—EXISTS, MAGISTRATE SIGNS AND ISSUES WARRANT(though officer has usually prepared warrant)

How much attention given to above by magistrates????

Officer admit to magistrate shopping for those who readily give warrants

SIMILAR PROCESS FOR ARREST WARRANTS

2.WAS WARRANT PROPER???

A. ISSUED & SIGNED BY NEUTRAL & DETACHED MAGISTRATE??

Principal issue----detachment—no stake in outcomeLo-Ji Sales (1979)

12

Page 13: Crim Pro Outline

Magistrate who left most of the “to be seized” portion of the warrant blank and went with the officers seizing a total of 900 pornographic materials violated the amendment by not remaining neutral and detached.

B. PARTICULARITY SUFFICIENT???

FROM 4TH--- Warrants……”particularly describing the place to be searched and the persons or things to be seized.”

Response to general warrants in colonial times

Purpose: To permit executing officer to be reasonably certain of executing warrant correctly.

Validity cannot hinge on what found but must rely on what info provided at time.

Andreson v. Maryland (1976)The particularity requirement is meant to prevent general searches. After a list of things to be searched the phrase “together with other fruits, instrumentalities, and evidence of the crime at this time unknown” was okay because coming as part of the same sentence that included the long list of specific items must be read as authorizing only the search for and seizure of evidence relating to the particular crime under investigation.

Defendant suspected of committing fraud in connection with sale of Lot 13T

Investigators applied for warrant to search his law office(assume affidavit established probable cause)

Language in warrant—Search for particular documents pertaining to sale and conveyance of Lot 13T.......“Together with other fruits, instrumentalities and evidence of crime at this time unknown.”

Groh v. Ramirez (2004)The fact that the application adequately describes the things to be seized does not save the warrant from its facial invalidity. The Fourth Amendment by its terms requires particularity in the warrant, not in the supporting documents. The Court does not say that the Fourth Amendment forbids a warrant from cross-referencing other documents. Most Courts of Appeals have held that a court may construe a warrant with reference to a supporting application or adffidavit if the warrant uses appropriate words of incorporation, and if the support document accompanies the warrant.

MUST COPY OF WARRANT BE PRESENTED TO “SEARCHEE?”

According to majority in United States v. Grubbs (547 U.S. 90, 2006) . –not required by 4th Amendment.

Also–Baranski v. 15 Unknown Agents of ATF, 401 F.3d 419 (6th Circ. 2006)

3. WAS WARRANT REASONABLY EXECUTED???

13

Page 14: Crim Pro Outline

A. EFFECT OF MISTAKES IN EXECUTION

Should evidence be suppressed if officer makes mistake and does something like search the wrong house????

Maryland v. Garrison (1987)Information that becomes available to officers immediately before or during the execution of a warrant may require them to cease or narrow their search, notwithstanding the dictates of the warrant. The officers erroneously entered Garrison’s premises; only after they discovered contraband there did they realize the mistake. They immediately ceased the search. The Court upheld the validity of the erroneous warrant and its execution.

WHEN MISTAKE MADE, WHETHER OR NOT EXECUTION OF WARRANT IS REASONABLE DEPENDS ON WHETHER THE OFFICERS’S FAILURE TO REALIZE THE MISTAKE WAS OBJECTIVELY UNDERSTANDABLE AND REASONABLE UNDER THE CIRCUMSTANCES AND THE REASONABLENESS OF THE OFFICER’S ENSUING ACTIONS.Dissent----Difficult to imagine that in the initial security sweep, a reasonable officer would not have discerned that 2 apartments were on the 3rd floor, realized his mistake and then confined ensuing search to McWebb’s residence.

AREA THAT CAN BE SEARCHED AND WHAT CAN BE SEARCHED IN THAT AREA DEPENDS ON THE SCOPE OF THE WARRANT AND WHAT’S LISTED IN THE WARRANT.(Note 7---p. 208)

If warrant is for house and lists illegal shotguns, can police searchclosets in the house???

small boxes and containers in the house????

“The same meticulous investigation which would be appropriate in a search for two small cancelled checks could not be considered reasonable where agents are seeking a stolen automobile or an illegal still.”

B. MANNER OF EXECUTION

Under common law, warrant could only be executed during daylight hours and officers had to “knock and announce” their presence.

What are the reasons for the “knock and announce” rule??

14

Page 15: Crim Pro Outline

WILSON V. ARKANSAS

Court held that 4th Amendment prohibition on unreasonable searches, contains implicit knock and announce rule but n 1992, Sharlene Wilson sold illicit narcotics to undercover agents of the Arkansas state police. Police officers then applied for and obtained warrants to search Ms. Wilson's home and to arrest her. When the police arrived, they found the main door to Ms. Wilson's house open. The officers opened the unlocked screen door and walked in, identified themselves as police officers, and said that they had a warrant. Ms. Wilson's attorney filed a motion to suppress the evidence seized during the search, claiming it was invalid on the grounds that the officers had failed to "knock and announce" before entering. Does the Fourth Amendment's reasonable search and seizure clause require police officers to knock and announce their presence before entering a private residence?Yes. A unanimous Court held that the common-law "knock-and announce" principle forms a part of the Fourth Amendment reasonableness inquiry. "Given the longstanding common-law endorsement of the practice of announcement, and the wealth of founding-era commentaries, constitutional provisions, statutes, and cases espousing or supporting the knock-and-announce principle,...the Amendment's Framers thought that whether officers announced their presence and authority before entering a dwelling was among the factors to be considered in assessing a search's reasonableness." Countervailing law enforcement interests, such as officer safety, may, however, establish the reasonableness of an unannounced entry.

Richards v. Wisconsin (1997)

Police in Madison, Wisconsin, suspected Steiney Richards of drug possession, but failed to receive a magistrate's authorization for a "no-knock" entry into his hotel room. Instead, they obtained a conventional search warrant requiring them to knock on Richards' door and identify themselves as officers prior to resorting to forcible entry. After arriving on the scene, an officer knocked on Richards' door identifying himself as a hotel custodian. When Richards opened the door, he saw a uniformed officer and quickly slammed it shut. The officers broke through the door, grabbed Richards while trying to escape, and found cocaine and cash in his bathroom. At trial, Richards challenged the constitutionality of the officer's search but was denied. On appeal, Wisconsin's Supreme Court affirmed and the Supreme Court granted certiorari. Did the officers' use of deception and force, in order to gain entry into Richards' hotel room, violate the Fourth Amendment's protection against illegal search and seizures? No. After noting the general importance of following conventional "knock-and-announce" procedures, the Court held that in those circumstances when police have good reason to suspect that announcing their presence and intentions may be dangerous, futile, or result in the destruction of evidence, a "no-knock" entry is justified. The Court added that by immediately closing the door after witnessing the officers outside it, Richards gave police sufficient justification for breaking into his room - especially considering the disposable nature of the substances they were seeking.

U.S. v. Banks (2003)Officers armed with a warrant to search Banks’s apartment for cocaine knocked loudly, announced their purpose, and waited 15 to 20 seconds before breaking the door with a battering ram and executing the warrant. The court held that 15 to 20 seconds without a response is a reasonable time so that police could suspect that cocaine would be gone if they reticent any longer because it was drugs but in other incidences they may need to wait longer.

Same answer in case where no-knock entry was approved by judge and police broke window to get in

15

Page 16: Crim Pro Outline

United States v. Ramirez, 523 U.S.65 (1998)—broke window

YES, but excessive or unnecessary destruction of property may violate 4th.

REASONABLENESS the key.

BUT---- EVEN IF VIOLATION OF KNOCK AND ANNOUNCE, UNDER FEDERAL LAW, EVIDENCE NEED NOT BE SUPPRESSED.

Illinois v. MacArthur (2001)Officers already on the scene suspect MacArthur of having pot. They left an officer behind while they went to get a warrant to make sure MacArthur didn’t reenter the home. Held that even though they had effectively seized defendant when they wouldn’t let him go back in the trailer, the warrant was upheld because he would have destroyed evidence.

Ybarra v. Illinois (1979)The police obtained a valid warrant to search a bar and Greg the bartender for evidence of drugs. While one officer searched the bartender another searched some of the customers and found heroin. The court held that the warrant did not authorize that search. There needed to be independent probable cause as well as some justification for conducting a search without a warrant.

Michigan v. Summers (1981)A warrant to search a residence for contraband founded on probable cause implicitly carries with it the limited authority to detain the occupants of the premises while a proper search is conducted.

Muehler v. Mena (2005)The right to detain under Summers includes the right to use reasonable force to secure and maintain detention of the occupant. They can handcuff if necessary.

Los Angeles County v. Rettele, 127S.Ct. 1989 (2007)

Investigation of 4 African-Americans suspected in a fraud and identity theft crime ring. One had registered a 9Ml. Glock handgun

Police get warrant to search 2 houses for the suspects and for documents and computer files as well as search suspects.

Had various info to establish PC that they resided at one of houses but did not know that one house had been sold to Rettele, a white man, who had been living there for 3 months.

Execute warrant, go in bedroom with guns drawn and order Rettele and wife out of bed....both are nude...wouldn’t let him put on sweatpants or her wrap sheet around her.

1 or 2 minutes later, allowed to dress–police realize they’ve made a mistake, apologize and leave.

Retteles file 1983 civil suit alleging warrant obtained in reckless fashion and search conducted (warrant

16

Page 17: Crim Pro Outline

executed) in unreasonable manner.

District Court granted Defendants Motion for Summary Judgment

9th Circuit reversed.

Do you think the Supreme Court should affirm the 9th Circuit??

WARRANT REQUIREMENT FOR IN-HOME ARRESTS

IN-HOME ARREST IS INTRUSION ON AREA TRADITIONALLY GIVEN HIGHEST 4TH AMENDMENT PROTECTIONIN PAYTON V. NEW YORK, 100 S.CT. 1371 (1980), SUPREMES HELD THAT IN THE ABSENCE OF EXIGENT CIRCUMSTANCES, 4TH AMEND PROHIBITS LEO FROM ARRESTING DEFENDANT IN HIS HOME WITHOUT AN ARREST WARRANT. RULE INTENDED TO PROTECT PERSON FROM HAVING HOME SEARCHED WITHOUT PROBABLE CAUSE TO ARREST. IF NO WARRANT, ARREST ITSELF IS NOT VIOLATIVE OF 4TH (AS LONG AS THERE IS PROBABLE CAUSE) BUT INSTEAD, SEARCH OF HOME WOULD VIOLATE 4TH

Example---Police go to Smith’s home to arrest him for a robbery. They have probable cause to believe he committed the robbery but do not get a warrant. While in his home effecting the arrest, police seize illegal guns that they discover. While Smith’s arrest does not violate the 4th, the guns would be inadmissible because police were in Smith’s home without a warrant.THE ARREST WARRANT CARRIES AUTHORITY TO SEARCH FOR SUSPECT ANYWHERE IN HOUSE WHERE HE MAY BE FOUND. So, in above example, if police had obtained an arrest warrant for Smith, they would have had authority to be in his house and items discovered during such “search” would be admissible if their seizure complies with rules regarding “plain view.” If they find suspect immediately upon entering home, arrest warrant does not give them authority to search other parts of home. To do so, they would need a search warrant or an exception to the warrant requirement. TO ENTER SUSPECT’S HOME WITH ARREST WARRANT, LEO MUST HAVE REASON TO BELIEVE THAT SUSPECT IS AT HOME. (“Reason to believe” equal to or less than probable cause---Circuit split)IF LEO HAS VALID SEARCH WARRANT, ARREST OF SUSPECT IN HOME DURING COURSE OF THE SEARCH IS VALID AS LONG AS THERE IS PROBABLE CAUSE TO ARREST….DON’T ALSO NEED ARREST WARRANT.

AN ARREST WARRANT IS NOT SUFFICIENT TO PROTECT THE PRIVACY INTERESTS OF A 3RD PARTY. IN STEAGALD V. U.S., 101 S.CT.1642, THE SUPREMES HELD THAT IN ABSENCE OF AN EXCEPTION TO THE WARRANT REQUIREMENT, LEO MUST OBTAIN A SEARCH WARRANT TO LOOK FOR A SUSPECT(AND ARREST THEM) IN THE HOME OF A THIRD PARTY. TO GET THE WARRANT, LEO WOULD HAVE TO ESTABLISH PROBABLE CAUSE THAT SUSPECT IN THAT HOME.HOWEVER, ONLY A RESIDENT OF THE HOME (ONE HAVING REP) WOULD HAVE STANDING TO OBJECT TO THE LACK OF A SEARCH WARRANT. AN ARRESTEE HAVING NO REP IN HOME WOULD NOT BE ABLE TO CONTEST THE LACK OF A SEARCH WARRANT.Example---Police go to Carter’s home to arrest Smith, who has stopped by Carter’s for a brief visit. They have an arrest warrant for Smith and reason to believe he is at Carter’s home. Despite Carter’s objection, the police

17

Page 18: Crim Pro Outline

go into his house to arrest Smith. While arresting Smith, police discover marijuana in Carter’s home and Carter is charged with possession. Carter has standing to contest the entry of the marijuana into evidence but, if Smith were charged with possession, he would not. Further, Carter’s Motion to Suppress should be granted because police did not have a search warrant.

In the above example, assume that police had probable cause to believe that Smith is at Carter’s home and based on such, obtained a search warrant to search Carter’s home for Smith. After obtaining the search warrant, police go to Carter’s home and, while in Carter’s home to arrest Smith, police observe the marijuana in plain view. In this scenario, the Motion to Suppress the marijuana should be denied.

SEARCH INCIDENT TO ARREST (SIA)

Chimel v. California (1969)

Local police officers went to Chimel's home with a warrant authorizing his arrest for burglary. Upon serving him with the arrest warrant, the officers conducted a comprehensive search of Chimel's residence. The search uncovered a number of items that were later used to convict Chimel. State courts upheld the conviction.In a 7-to-2 decision, the Court held that the search of Chimel's house was unreasonable under the Fourth and Fourteenth Amendments. The Court reasoned that searches "incident to arrest" are limited to the area within the immediate control of the suspect. While police could reasonably search and seize evidence on or around the arrestee's person, they were prohibited from rummaging through the entire house without a search warrant. The Court emphasized the importance of warrants and probable cause as necessary bulwarks against government abuse.

U. S. V. Robinson (1973)

A police officer pulled over and arresting Robinson for operating an automobile without a valid permit. The officer then frisked Robinson and discovered a crumpled cigarette package containing fourteen vials of heroin in his pocket. Did the officer's search violate the Fourth Amendment? The Court upheld the search. Distinguishing between searches done to discover concealed weapons and those conducted coextensive with an arrest, Justice Rehnquist argued since the officer did not conduct the search in an abusive or extreme manner, and because he acted consistent with the authority vested in a police officer when making an arrest, his actions were legitimate.

Inventory Searches

Inventory searches are performed before one is jailed to protect the other prisoners from contraband. It is a constitutional search and one can be charged for what they find.

APPLICATION OF SIA TO AUTOMOBILES

WHAT ARE PARAMETERS OF SIA SEARCH WHEN CUSTODIAL ARREST OF OCCUPANTS OF AN AUTOMOBILE??

18

Page 19: Crim Pro Outline

New York v. Belton (1981) The United States Supreme Court held that when a policeman has made a lawful custodial arrest of the occupant of an automobile, he may, as a contemporaneous incident of that arrest, search the passenger compartment of that automobile. Therefore, Belton extended the so-called “Chimel rule” of searches incident to a lawful arrest, established in Chimel v. California (1969), to vehicles. The Supreme Court sought to establish bright line rules to govern vehicle search incident to eliminate some confusion in the cases. The court held that when a policeman has made a lawful custodial arrest of the occupant of an automobile, he may, as a contemporaneous incident of that arrest, search the passenger compartment of that automobile.It follows from this conclusion that the police may also examine the contents of any containers found within the passenger compartment, for if the passenger compartment is within reach of the arrestee, so also will containers in it be within his reach. United States v. Robinson, supra; Draper v. United States, 358 U.S. 307 . Such a container may, of course, be searched whether it is open or closed, since the justification for the search is not that the arrestee has no privacy interest in the container, but that the lawful custodial arrest justifies the infringement of any privacy interest the arrestee may have. Thus, while the Court in Chimel held that the police could not search all the drawers in an arrestee's house simply because the police had arrested him at home, the Court noted that drawers within an arrestee's reach could be searched because of the danger their contents might pose to the police.Thus, under Belton, the entire passenger compartment of an automobile is subject to search under the search incident doctrine even if the arrestee is out of the car.

A nexus is required between the vehicle and the person arrested with or in the vehicle prior to the arrest

Knowles v. Iowa (1998)After stopping him for speeding, an Iowa police officer issued Patrick Knowles a citation and conducted a full search of his car without probable cause or Knowles' consent. When his search turned up a "pot pipe" and some marijuana, the officer arrested Knowles on state drug charges. Knowles challenged these on grounds that because he was not arrested at any time prior to the search, the search was unconstitutional. On appeal from consecutive adverse rulings in lower courts, the Supreme Court granted Knowles certiorari. Can a search of a stopped vehicle, that occurs prior to the driver's arrest, be sustained under the "search incident to arrest" exception that permits officers to search stopped vehicles without first obtaining a search warrant? No. In a unanimous opinion that Court held that full stopped-car searches can only be conducted when the safety of the officers is at risk. One significant indication of such danger is when an officer arrests the subject as a reaction to possible or actual threat. In the present case, no serious danger accompanied the stop of Knowles car as evidenced by the officer's initial decision not to arrest Knowles or even issue him a ticket. As such, regardless of its uncovered contents, the subsequent search violated the "search incident to arrest" power and the Fourth Amendment's prohibition again unlawful search and seizures.

Atwater v. Largo Vista (2001)Woman and her two children who weren’t wearing seatbelts was thrown in jail.HOLDING-----A STATE STATUTE ALLOWING FOR A CUSTODIAL ARREST FOR MINOR OFFENSES IS NOT UNREASONABLE AND THUS DOES NOT VIOLATE 4TH AMENDMENT.MOST COURTS INTERPRETED BELTON AS APPLICABLE TO ALL SITUATIONS WHERE OFFICER INITIATES CONTACT WITH AN OCCUPANT OF A VEHICLE AND THEN MAKES ARREST.

BUT—WHAT IF PERSON HAS PARKED AND GOTTEN OUT OF CAR??

Thornton v. U.S. (2004)

19

Page 20: Crim Pro Outline

EXTENDED BELTON---IF ARRESTEE A RECENT OCCUPANT OF VEHICLE, POLICE MAY SEARCH VEHICLE INCIDENT TO ARREST

1AUTOMOBILE EXCEPTION

Chambers v. Maroney (1970)The United States Supreme Court applied the Carroll doctrine in a case with a significant factual difference—the search took place after the vehicle was moved to the stationhouse. The search was thus delayed and did not take place on the highway (or street) as in Carroll. After a gas station robbery, a vehicle fitting the description of the robbers' car was stopped. Inside were people wearing clothing matching the description of that worn by the robbers. They were arrested, and the car was taken to the police station where it was later searched. The Court first held that the search could not be sustained as a search incident under Preston and Dyke. It quoted at length from Carroll that a search of a movable vehicle is treated differently under the Fourth Amendment because the mobility of the vehicle alone can easily defeat the warrant requirement.[2] If there is probable cause to believe the vehicle contains criminal evidence and there exist exigent circumstances where the vehicle can be removed from the jurisdiction, a warrantless search would be reasonable.[3] It made no constitutional difference here that the search followed the seizure because the probable cause which developed on the street still existed at the stationhouse.[4] For this purpose, it is significant to note that the automobile exception and the search incident doctrine are quite different

Coolidge v. New Hampshire (1971)The state sought to justify the search of Coolidge's car on three theories: automobile exception, search incident, and plain view. Assuming there was probable cause for the search of Coolidge's car, the Court said "the application of the Carroll doctrine to these facts would extend it far beyond its original rationale." It could not be shown that it was not practical under the circumstances to obtain a warrant because of exigent circumstances and because "the opportunity to search is fleeting." Here, the police knew for quite some time that the vehicle was possible evidence, and the suspect was cooperative. Also, the objects being searched for were scientific evidence and not contraband. Since Coolidge was arrested in the house and his car was towed in, he had no access to it to destroy evidence. The plurality held that the Carroll doctrine was inapplicable because there were no exigent circumstances justifying dispensing with the warrant requirement.

Carroll doctrine: a vehicle could be searched without a warrant if there was probable cause to believe that evidence is present in the vehicle, coupled with exigent circumstances to believe that the vehicle could be removed from the area before a warrant could be obtained.

TWO PRONGED TEST FOR AUTOMOBILE EXCEPTION TO WARRANT REQUIREMENT

1. MOTOR VEHICLE

2. PROBABLE CAUSE TO BELIEVE CONTRABAND OR EVIDENCE OR FRUITS OF CRIME WILL BE FOUND IF BOTH PRONGS, SEARCH WITHOUT WARRANT IS REASONABLE UNDER 4TH UNLESS A LONG DETENTION (LIKE COOLIDGE)

ENTIRE VEHICLE CAN BE SEARCHED INCLUDING THE TRUNK OR THE BED OF A TRUCK

20

Page 21: Crim Pro Outline

California v. Carney (1985)A motor home is a vehicle subject to the automobile exception to the warrant requirement because it is readily moveable.

PROBLEM

John’s motor home, which is licensed for road travel, is parked at a campsite on top of a steep hill in a state park. He has it plugged into the electrical outlet that is provided. He and his two sons have been camping there for a week. A state conservation officer walks by and smells what he recognizes as the smell of burnt marijuana. The officer knocks on the door to the motor home but since John and his sons are out hiking, the officer gets no response. Can the officer search the motor home without a warrant?

CONTAINERS IN CARS

WHAT WITHIN THE VEHICLE MAY BE SEARCHED???

UNITED STATES V. ROSS (1982)Police had probable cause to believe that someone was selling drugs from the trunk of their car at a specified location. They searched the truck and found a paper bag which they opened and found heroin. The court held that when the police have probable cause to search a car without a warrant, they may also search any container found during the car search that is large enough to hold the evidence for which they are looking.

CALIFORNIA V. ACEVEDO (1991)

California police officers saw Charles Acevedo enter an apartment known to contain several packages of marijuana and leave a short time later carrying a paper bag approximately the same size as one of the packages. When Acevedo put the bag in the truck of his car and began to drive away, the officers stopped the car, searched the bag, and found marijuana. At his trial, Avecedo made a motion to suppress the marijuana as evidence, since the police had not had a search warrant. When the trial court denied his motion, Acevedo pleaded guilty and appealed the denial of the motion. The California Court of Appeal reversed the trial court, ruling that the marijuana should have been suppressed as evidence. The Supreme Court had ruled previously that officers can thoroughly search an automobile if they have probable cause to believe there is evidence somewhere in the vehicle (U.S. v. Ross), and also that officers need a warrant to search a closed container (U.S. v. Chadwick). The California Court of Appeal decided that the latter case was more relevant. Since the officers only had probable cause to believe the bag contained evidence - not the car generally - they could not open the bag without a search warrant. The California Supreme Court denied review, but the Supreme Court granted the State's petition.Under the Fourth Amendment, may police conduct a warrantless search of a container within an automobile if they have probable cause to believe that the container holds evidence? Yes. In a 6-3 decision authored by Justice Harry Blackmun, the Court reversed the Court of Appeal and ruled that the "automobile exception" to the Fourth Amendment's general search-warrant requirement is broad enough to cover a situation where the police only have probable cause to believe there is evidence in a specific movable container within the car. The Court noted that the warrant requirement previously had depended on a "curious line between the search of an automobile that coincidentally turns up a container and the search of a container that coincidentally turns up in an automobile." In place of that uncertain distinction, the Court adopted a single rule: "The police may search an automobile and the containers within it where they have probable cause to believe contraband or evidence is contained."

21

Page 22: Crim Pro Outline

INVENTORY SEARCHES

TEST

Impoundment must be lawful

Must be of routine nature and pursuant to standard procedures

In Florida and most states, cannot be mere subterfuge for criminal investigation

FL—Must have written policy and in order to search containers, the policy must specifically cover containers.

2nd DCA----for State to prevail at suppression hearing, copy of policy must be introduced.

PLAIN VIEW “EXCEPTION” ACTUALLY AFFECTS WHAT ITEMS MAY LAWFULLY BE SEIZED

HORTON V. CALIFORNIA (1990)Case that developed the plain view doctrine in criminal procedure. The Supreme Court had already recognized the doctrine in Coolidge v. New Hampshire (1971) and in Arizona v. Hicks (1987), but expanded it in Horton. Test:That expansion included a three-part test, requiring that the police officer finding evidence in plain view be:

1. lawfully present at the place where the evidence can be plainly viewed,2. the officer must have a lawful right of access to the object, and3. the incriminating character of the object must be “immediately apparent.

Horton also eliminated the inadvertent discovery requirement.

Arizona v. Hicks (1987)

Officer was found to have acted unlawfully. While investigating a shooting, the officer moved, without probable cause, stereo equipment to record the serial numbers. The plain view doctrine has also been expanded to include the sub doctrines of plain feel, plain smell, and plain hearing.

NOTE THAT THE COURT DISTINGUISHES POLICE ACTIONS HERE FROM A MERE CURSORY INSPECTION AND NOTES THAT A CURSORY INSPECTION ISN’T EVEN A SEARCH BECAUSE IT INVOLVES MERELY LOOKING AT WHAT IS ALREADY EXPOSED TO PUBLIC VIEW.

REQUIREMENTS FOR “REASONABLE” PLAIN VIEW SEIZURE

1. Officer’s original intrusion or location must be lawful----i.e.—no violation of 4th Amendment to arrive at vantage point.

Example---if officer goes to George’s house to arrest John and does not have a search warrant, his entry into George’s house to search for John is

22

Page 23: Crim Pro Outline

“unreasonable” under the 4th Amendment (assuming no other exception to the warrant requirement) Therefore the plain view doctrine would not justify the seizure of items seen while searching for John.

2. Officer must lawfully be in a position to seize the item---i.e.—item must be observed while officer is confining activitiesto the permissible scope of the intrusion.

Example---If, in above example, Officer does have a search warrant tosearch George’s house for John, the plain view doctrine would not apply to the seizure of items from a closed drawer.

3. It must be immediately apparent that the item that is seized is contraband or evidence or fruits of a crime---or in other wordsthere must be probable cause to believe the item is contraband,etc. There cannot be further examination or search to make thisdetermination.

(Example----Arizona v. Hicks, the stereo components case)

Under federal law, the discovery does not have to be inadvertent, which means even if officer thought he might find the items that are in plain view before he went into the house, the plain view doctrine would still apply, as long as above three prongs are satisfied.

CONSENT EXCEPTION

AS BOOK POINTS OUT, THERE ARE LEGIONS OF CONSENT SEARCHES…….

Schneckloth v. Bustamonte (P. 310)An officer on a routine patrol at 2:30 in the morning observed an automobile with a broken headlight. He stopped the car, and the driver was unable to produce a driver's license. There were five other passengers, and only one could produce a license. The officer asked that person (who said that the car was his brother's) if he could search the car. The person allowed him to search, and the officer found three stolen checks. Bustamonte (D) was convicted after these checks were admitted into evidence at his trial. The California Court of Appeal affirmed his conviction. D sought a writ of habeas corpus in a federal district court. He was denied, but the Court of Appeals set aside the district court's order. Schneckloth (P) appealed.Issue: Must the State always bear the burden of proof that consent to a search was voluntarily given?Holding: No they just must show that consent was not coerced.Procedural Posture: Petitioner sought review from a decision of the United States Court of Appeals for the Ninth Circuit, which vacated order denying a writ of habeas corpus to respondent and remanded case for further proceedings, holding that consent could not be found solely from the absence of coercion and a verbal expression of assent. The Court reversed the decision of the court below, reinstating the affirmation of respondent's conviction and denial of writ of habeas corpus to respondent, on the grounds that the determination of voluntariness did not require proof of knowledge of a right to refuse as the sine qua non of an effective consent to a search.Reasoning: Based on the totality of the circumstances (namely: Characteristics of the accused, and Details of the interrogation), when the subject of a search is not in custody and the State attempts to justify a search on the basis of his consent the state must demonstrate that the consent was in fact voluntarily given, and not the result of duress or coercion, express or implied. The Due Process Clause does not require the state to prove that the

23

Page 24: Crim Pro Outline

defendant knew he had a right to refuse to answer questions. His state of mind and the police's failure to advise him of his rights are certainly factors, but are not in themselves determinative. In this case, there is no evidence of any inherently coercive tactics.

VOLUNTARINESS DEPENDS ON TOTALITY OF CIRCUMSTANCES

What subjective factors are relevant??What objective factors are relevant??Why do guilty people consent to searches???

Ohio v. Robinette (1996)Although knowledge is a factor to be taken into account in voluntariness analysis, there is no categorical requirement that police officers inform detainees that they are free to go before a consent to search may be deemed voluntary.

Bumper v. North Carolina (1968)

Police told 66 year old widow they had a warrant but they didn’t. She consented to search. The court held that because she was in effect told that they had a right to search that even though she told them to go ahead, her consent was not voluntarily and freely given.

TEST FOR SCOPE---What would the reasonable person have understood from theexchange between the officer and the suspect??

CAN WITHDRAW CONSENT----Withdrawal of consent, standing alone, cannotsupport finding of probable cause or reasonable suspicion

SAYING NOTHING DOES NOT EQUAL CONSENT

TWO PRONGED TEST FOR VALID CONSENT

1. CONSENT MUST BE VOLUNTARY

----STATE MUST SHOW THAT, BASED ON TOTALITY OF CIRCUMSTANCES, CONSENT WAS VOLUNTARY----KNOWING WAIVER OF RIGHT NOT REQUIRED (Schneckloth v. Bustamonte)

FACTORS CONSIDERED IN DECIDING WHETHER VOLUNTARY (NOT EXHAUSTIVE)

Characteristics of accused-----age, education, physical and mental condition, etc.Details of police-citizen interactionPresence of coercive police procedures (weapons, # of officers, threats, etc.)Voluntariness of custodial statusExtent and level of suspect’s co-operation with police

24

Page 25: Crim Pro Outline

Suspect’s awareness of right to refuseSuspect’s belief that no evidence will be foundPolice deception and promises

Knowledge of what consent is for (Ferguson v. City of Charleston)

2. PERSON GIVING CONSENT MUST HAVE AUTHORITY TO CONSENT

A. ACTUAL AUTHORITY

1. IF MUTUAL USE OF PROPERTY??

Doctrine of common authority

LANDLORD LACKS AUTHORITY TO CONSENT TO SEARCH OF TENANT’S APARTMENT

SAME WITH HOTEL MANAGER & GUEST

25

Page 26: Crim Pro Outline

Georgia v. Randolph (2006)

Scott Randolph was arrested for drug possession after police found cocaine in his home. The police did not have a warrant to search the home, but Randolph's wife consented to the search. Randolph was also present at the time of the search, however, and objected to the police request. At trial, his attorney argued that the search was unconstitutional because of Randolph's objection, while the prosecution argued that the consent of his wife was sufficient. The trial court ruled for the prosecution, but the appellate court and Georgia Supreme Court both sided with Randolph, finding that a search is unconstitutional if one resident objects, even if another resident consents. Can police search a home when one physically present resident consents and the other physically present resident objects? No. In a 5 to 3 decision, the Supreme Court held that when two co-occupants are present and one consents to a search while the other refuses, the search is not constitutional. Justice David Souter, in the majority opinion, compared the reasonableness of such a search to a more casual interaction. Souter wrote, "it is fair to say that a caller standing at the door of shared premises would have no confidence that one occupant's invitation was a sufficiently good reason to enter when a fellow tenant stood there saying, 'stay out.' Without some very good reason, no sensible person would go inside under those conditions." A police search in such circumstances, Souter wrote, would therefore not meet the reasonableness requirement of the Fourth Amendment.

U.S. v. Hudspeth, 518 F.3d, 954

Discovered porn on Mr. Hudspeth’s work computer.

Cops asked for permission to search home computer

Mr. H refused to give them permission–he’s arrested

3 cops go to his house and ask Mrs. H for permission to searchhome computer—tell her their concerns but don’t tell her that husband said “no”

In response to her inquiry, they told her that if she didn’t consent they would leave officer there (3 kids) and go get warrant

She couldn’t reach attorney and finally consented.

PANEL OF 8TH CIRCUIT IN 2006 FOUND HER CONSENT VOLUNTARY, BUT NOT VALID, BECAUSE IT COULD NOT OVERRULE HIS DENIAL. HOWEVER, 8TH CIRCUIT, EN BANC IN 2008, FOUND THAT HIS OBJECTION DID NOT PROHIBIT POLICE FROM ENTERING ON THE BASIS OF HER CONSENT.

B. APPARENT AUTHORITY

26

Page 27: Crim Pro Outline

Looks at situation from perspective of police officer----would a reasonable police officer under the circumstances have believed that the 3rd party had actual authority to consent??

Illinois v. Rodriguez (1990)Whether a warrantless search conducted pursuant to third party consent violates the Fourth Amendment when the third party does not actually possess common authority over the premises. In a 6-3 decision authored by Justice Antonin Scalia, the Court held that such searches are valid if, at the time of the search, the authorities "reasonably believe" the third party possesses common authority over the premises.

In reaching its decision, the Court noted that "reasonableness," not consent, is the touchstone of Fourth Amendment jurisprudence; the Constitution only prohibits "unreasonable" searches and seizures. Therefore, the constitutional validity of a police determination of consent to enter is not judged by whether the police were correct in their assessment, but by whether, based on the facts available at the moment, it was reasonable to conclude that the consenting party had authority over the premises.

PROBLEM

The Gutmans are driving rented car with FL plates on interstate in N.M.

Passed LEO and partner—they notice that Mr. G. is not wearing his seat belt, which, in New Mexico is a traffic violation.

Followed for 3 miles, then pulled over.

LEO and partner approach car. They determine that license and rental agreement o.k.–had all info needed to write citation but continued to question

Asked them about travel plans and Mr. G said they were going to Las Vegas

Mrs. G. very pregnant, perspiring and avoided eye contact

LEO suspects drugs or contraband in car

While one officer is questioning, the other walks around the car, peering into the windows.

Asks them if drugs or contraband in car—Gutmans say no.

After 15 minutes of conversation, hands Mr. G. a “consent to search” form

Joe signed consent to search form.

27

Page 28: Crim Pro Outline

Joe is from Mexico and obtained a green card in U.S. He has been in U.S. 10 years and has a flourishing gardening business, but has only a 10th grade education.

Officer searches—finds drugs.

Gutmans file Motion to Suppress.

Arguments????

How should court rule???

1 “SPECIAL NEEDS” SEARCHES AND SEIZURES

SEARCHES CONDUCTED FOR NON-CRIMINAL INVESTIGATION- RELATED PURPOSE (regulatory or administrative

THE GOVT’S COMPELLING NEED REPLACES NEED FOR EITHER WARRANT, PROBABLE CAUSE AND/OR REASONABLE SUSPICION—REQUIREMENTS EITHER MODIFIED OR ELIMINATED TWO-STEP ANALYSIS

1. Is the govt. purpose truly regulatory?? (vs. criminal law enforcement)

2. Balancing interest of government against the individual privacy interest infringed upon. (REASONABLENESS BALANCING)

Factors affecting balancing

1. Weight of state’s (public) interest (gravity of concern)

2. Effectiveness of chosen means of achieving goal (degree to which public interest advanced)

3. Availability of less restrictive alternative means

4. Length of time regulation in existence / acceptance by society

5. Degree of individual interest —car or business vs. home ----children vs. adults

6. Degree to which interest is invaded -----long stop vs. short stop ------patdown search vs. strip search

28

Page 29: Crim Pro Outline

7. Limitations on discretion of govt actors (clear rules or policies)

CATEGORIES OF SEARCHES (not exhaustive)

Security searches (airports, subways, govt buildings, etc.)

Health and safety inspections

Inspections of pervasively regulated industries

Drug testing

Searches of probationers and parolees

Border Searches

Roadblocks

Inventory Searches HEALTH AND SAFETY INSPECTIONS

Do health inspectors need warrant based on individualized probable cause to go into Al’s Pizza and make sure in compliance with various regulations??

Why or why not??

Rule from Camara v. Municipal Court: Needed administrative warrants to search based on probable cause (could just be that the building needed to be inspected). Today, warrantless searches are okay even in the absence of emergency or consent.

BURGER CRITERION RE: ADMINISTRATIVE SEARCHESOF PERVASIVELY REGULATED BUSINESSES

1. Substantial governmental regulatory interest

2. Warrantless inspection scheme is necessary to further the regulatory scheme.

PRIMARY PURPOSE MUST BE REGULATORY—NOT TO FURTHER LAW ENFORCEMENT INTERESTS

3. The inspection program, in terms of certainty and regularity of its application, provides a constitutionally adequate substitute for a warrant.

29

Page 30: Crim Pro Outline

DRUG TESTING

3 CATEGORIES—TREATED DIFFERENTLY

1. SEARCHES OF CHILDREN (STUDENTS)

2. ADULTS WORKING IN SAFETY SENSITIVE CONTEXT

3. ADULTS IN NON-SENSITIVE CONTEXT

STUDENTS

New Jersey v. T.L.ONeither the warrant requirement nor the probable cause requirement is necessary to search public school children. The court held that school searches are okay so long as 1) there are reasonable grounds---not necessarily probable cause in the criminal law context---for suspecting that the search will turn up evidence that the student has violated or is violating either the law or the rules of the school; and 2) once initiated, the search is not excessively intrusive in light of the age and sex of the student and the nature of the infraction.

Veronia School District v. Acton (1995)upheld the constitutionality of random drug testing regimen implemented by the local public schools in Vernonia, Oregon. Under that regimen, student athletes were required to submit to random drug testing before being allowed to participate in sports. During the season, 10% of all athletes were selected at random for testing. The Supreme Court held that although the tests were searches under the Fourth Amendment, they were reasonable in light of the schools' interest in preventing teenage drug use.

Board of Education v. Earls (2002)Upheld the constitutionality of mandatory drug testing by public schools of students participating in extracurricular activities. The legal challenge to the practice was brought by two students, Lindsay Earls and Daniel James, and their families against the school board of Tecumseh, Oklahoma, alleging that their policy requiring students to consent to random urinalysis testing for drug use violated the Fourth Amendment to the United States Constitution.The majority decision, written by Justice Clarence Thomas, with a concurring opinion by Justice Stephen Breyer, held that students in extracurricular activities had a diminished expectation of privacy, and that the policy furthered an important interest of the school in preventing drug use among students. This rationale was based on the precedent Vernonia School District 47J v. Acton, which allowed drug testing for athletes.

Despite there being little evidence of any drug problems amongst students involved in extracurricular activities, court applied reasonableness balancing and rationale from Veronia

30

Page 31: Crim Pro Outline

found that the testing was reasonable.

Safford United School District v. Redding (2009)Reasonable suspicion that 13 year old distributing illegal drugs(prescription strength ibuprofen)School official conducted strip search including searching underbra and underwearCOURT----Violation of 4th Amendmentgovt interest vs. intrusiveness of searchO.K. to search backpack and outer garments but this level of intrusion was excessive and not justified given the content of the suspicion

SUSPICIONLESS DRUG TESTING OF ADULTSSuspisionless drug testing of adults has been upheld in various circumstances: drug testing of personnel involved in train accidents, random drug testing of federal customs officers who carry weapons or are involved in drug interdiction; but Georgia’s requirement that candidates for state office pass a drug test did not fit within the closely guarded category of constitutionally permissible suspicionless searches.

In testing drug cases, the court considers the nature and immediacy of the government’s concerns regarding drug use. In some cases, the Court has found a compelling, substantial or important governmental or societal need for drug testing that could not be accomidated by application of ordinary probable cause or reasonable suspicion standards.

FERGUSON V. CITY OF CHARLESTON (2001)Instituted at public hospital trying to test pregnant mothers suspected of drug use. This did not fall under the special categorical exceptions and requires a search warrant.

Court said that the consent could not be considered voluntary because the women werenot aware that the urine samples were being turned over to law enforcement

MAJORITY HELD THAT ABSENT CONSENT OR EXIGENCY, THE WARRANTLESS URINE TESTING VIOLATED THE 4TH AMENDMENT

BORDER SEARCHES

RULE RE: BORDER SEARCHES

SUSPICIONLESS ROUTINE SEARCHES AT INTERNATIONAL BORDER OR AT AIRPORT WHEN ARRIVING ON INTERNATIONAL FLIGHT ARE REASONABLE.

WHY?? (APPLYING REASONABLENESS BALANCING)

Luggage searches and patdowns are considered routine—

31

Page 32: Crim Pro Outline

Other searches—depends on degree of invasiveness or intrusiveness associated with search.

EXAMPLES

U.S. v. Flores-Montano 541 U.S. 149, 124 S.Ct. 1582 (2004)

No reasonable suspicion needed to remove, disassemble and reassemble a vehicle’s fuel tank while conducting border search.

Reduced expectation of privacy in car

“While it may be true that some searches of property are so destructive as to require a different result, this was not one of them.”

U.S. v. Cortez-Rocha, 394 F.3d 1115 (9th Cir. 2005)

O.K. to cut open spare tires during border search (no r.s. necessary)

U.S. v. Arnold, 523 F.3d 941 (9th Cir. 2008)

4th Amendment does not require reasonable suspicion in order to browse through files stored on laptop computer of passenger on international flight coming through customs.

U.S. v. Whitted, 541 F.3d 480 (3rd Cir. 2008)

Defendant on cruise ship from St. Maarten–when docked at St. Thomas, customs officers (based on a tip) entered his stateroom to search for drugs.

Routine (no reasonable suspicion needed) or non-routine (r.s. needed) ????

HOLDING—Non-routine and R.S. necessary.

But–Court went on to say they had R.S.

U.S. v. Montoya de Hernandez, 473 U.S. 531 (1985)

Court upheld a 19-hour detention of a “balloon swallower” who flew to U.S. from Columbia. After initial questioning by customs agents, developed reasonable suspicion. Conducted patdown and strip search (female inspector)—stomach felt unusually firm plus she had on extra underwear. After several hours she “exhibited symptoms of discomfort consistent with heroic efforts to resist the usual calls of nature.” Got an order from a federal Magistrate authorizing a rectal exam.

32

Page 33: Crim Pro Outline

Court held that while search was non-routine, since at border, only needed reasonable suspicion.

“We hold that the detention of a traveler at the border, beyond the scope of a routine customs search and inspection, is justified at its inception if customs agents, considering all the facts surrounding the traveler and her trip, reasonably suspect that the traveler is smuggling contraband in her alimentary canal.”

RE: Whether she was protected by 4th----“having presented herself at the border for admission, and having subjected herself to the criminal enforcement powers of the federal government, respondent was entitled to be free from unreasonable search and seizure.”

AWAY FROM THE BORDER ITSELF??

With roving border patrols, the Supreme Court has determined that the agents need reasonable suspicion of criminal activity to detain the car occupants briefly (U.S. v. Brignoni-Ponce). But in U.S. v. Matinez-Fuerte, the court held that vehicle occupants may be stopped for questioning at fixed interior checkpoints without individualized suspicion of wrongdoing.

The distinction is based on two grounds. First, the subjective intrusion on the security of lawful travelers, their fear and surprise level, is appreciably less in the case of a fixed checkpoint stop. Second, agents at fixed checkpoints have less discretionary enforcement authority than roving agents: the location of the checkpoint is fixed, and they may only stop those that pass through it.

1979—Delaware v. Prouse

Supremes said cannot randomly stop drivers to check license and registration. However—noted that states were not precluded from developing methods for spot checks that involve less intrusion or do not involve unconstrained exercise of discretion by police officers.

Government interest was legitimate----means were not. Unfettered discretion too much of a privacy invasion.

Michigan v. Sitz (1990)DUI Checkpoints-- The Supreme Court held that Michigan had a "substantial government interest" to advance in stopping drunk driving, and that this technique was rationally related to achieving that goal (though there was some evidence to the contrary). The Court also held that the impact on drivers, such as in delaying them from reaching their destination, was negligible, and that the brief questioning to gain "reasonable suspicion" similarly had a negligible impact on the drivers' Fourth Amendment right from unreasonable search (implying that any more detailed or invasive searches would be treated differently). Applying a balancing test, then, the Court found that the Constitutionality of the search tilted in favor of the government.

City of Indianapolis v. Edmond (2000)

33

Page 34: Crim Pro Outline

Limited the power of law enforcement to conduct suspicionless searches, specifically, using drug-sniffing dogs at roadblocks. Previous Supreme Court decisions had given the police power to create roadblocks for the purposes of border security (United States v. Martinez-Fuerte), and removing drunk drivers from the road (Michigan Dept. of State Police v. Sitz). This decision stated that the power was limited to situations in which the search was "designed to serve special needs, beyond the normal need for law enforcement."

The Court drew a line on check point programs that followed Police v. Sitz (1990) "whose primary purpose" is "to detect evidence of ordinary criminal wrongdoing". The Court refused to "credit the 'general interest in crime control' as justification for a regime of suspicionless stops."

Illinois v. Lidster (2004) Police stopped Robert Lidster at a checkpoint set up to find information about a recent hit-and-run accident. Lidster was arrested, and later convicted, for drunk driving. Lidster successfully appealed his conviction to the Illinois Appellate Court. It relied on the U.S. Supreme Court's decision in Indianapolis v. Edmond (2000) holding that a checkpoint is unconstitutional if its only purpose is to uncover "ordinary criminal wrongdoing." The Illinois Supreme Court affirmed. Question: Does Indianapolis v. Edmond, which dealt with the Fourth and 14th Amendment prohibitions of unreasonable searches and seizures, prohibit checkpoints organized to question motorists about a previous offense and arrest motorists for drunk driving?Conclusion: No. In an opinion delivered by Justice Breyer, the Court held 6-3 that the Illinois checkpoint did not violate the Fourth Amendment's prohibition of unreasonable searches and seizures and was constitutional. It ruled that the checkpoint was reasonable because it advanced a "grave" public interest - "investigating a crime that had resulted in a human death" - and interfered minimally with Fourth Amendment liberty. The Court distinguished Illinois's "information-seeking" checkpoint from the "crime control" checkpoint struck down in Edmond. Justices Stevens, Souter, and Ginsburg - while agreeing that Edmond does not invalidate the Illinois checkpoint - dissented from the majority's decision granting constitutional approval to the checkpoint. They argued that the case should have been remanded to the Illinois courts.

WHICH OF THE FOLLOWING DO YOU BELIEVE WOULD BE UPHELD?

34

Page 35: Crim Pro Outline

MISSOURI CHECKPOINT OHIO CHECKPOINT

Trickery No trickery

Remote location Plan publicized

Drivers couldn’t see anything No indication that drivers couldn’tuntil stopped tell what was happening

Camaflouged officer approachedafter single officer stopped

Written guidelinesNo guidelines for officers for officers

No accountability Filmed by media

PROBLEM IN NOTE 5, p. 445

PROBLEM IN NOTE 6, p. 445

OTHER TYPES OF SEARCHES THAT HAVE BEEN UPHELD PURSUANT TO HIGH GOVT INTEREST OR NEED

Random, suspicionless searches of subway riders

Suspicionless searches at courthouses

Suspicionless searches of people entering stadiums (one recent caseout of 11th Circuit found consent)

1TERRY V. OHIO

held that the Fourth Amendment prohibition on unreasonable searches and seizures is not violated when a police officer stops a suspect on the street and searches him without probable cause to arrest, if the police officer has a reasonable suspicion that the person has committed, is committing, or is about to commit a crime.

For their own protection, police may perform a quick surface search of the person’s outer clothing for weapons if they have reasonable suspicion that the person stopped is armed. This reasonable suspicion must be based on "specific and articulable facts" and not merely upon an officer's hunch. This permitted police action has subsequently been referred to in short as a "stop

35

Page 36: Crim Pro Outline

and frisk," or simply a "Terry stop". The Terry standard was later extended to temporary detentions of persons in vehicles, known as traffic stops.

The rationale behind the Supreme Court decision revolves around the understanding that, as the opinion notes, "the exclusionary rule has its limitations." The meaning of the rule is to protect persons from unreasonable searches and seizures aimed at gathering evidence, not searches and seizures for other purposes (like prevention of crime or personal protection of police officers).

Until Terry, only seizure of person governed by the 4th was arrest

Terry expanded that.

Q----WHAT OBSERVATIONS DID OFFICER MCFADDEN MAKE???

Q—DID HE KNOW THEM OR HAVE ANY INFORMATION ABOUT THEM?

Q–WHAT ACTIONS DID OFFICER TAKE??? WHAT WAS HIS JUSTIFICATION??

MOTION TO SUPPRESS GUNS THAT WERE FOUND

ARGUMENT??

Q—DID (OR WOULD) ANY COURT FIND THAT THERE WAS PROBABLE CAUSE TO ARREST???

Q—DID OFFICER’S ACTIONS RISE TO AN ARREST???

Q—WAS TERRY SEIZED???

Q—WAS THERE A SEARCH???

SO—MUST DECIDE IF SEIZURE AND SEARCH WERE REASONABLE OR UNREASONABLE.

WHAT IS GENERAL TEST???? -----WHETHER OFFICER’S ACTION WAS JUSTIFIED AT ITS INCEPTION AND WHETHER IT WAS REASONABLY RELATED IN SCOPE TO THE CIRCUMSTANCES WHICH JUSTIFIED THE INTERFERENCE IN THE 1ST PLACE. BALANCING NEED TO SEARCH BASED ON SPECIFIC ARTICULABLE FACTS TOGETHER WITH INFERENCES FROM THOSE FACTS AGAINST THE INTRUSION. AFFECTED BY GOVT. INTEREST—WHAT IS GOVT. INTEREST HERE???

36

Page 37: Crim Pro Outline

Q--IN ORDER TO FRISK MUST OFFICER BE CERTAIN THAT PERSON ARMED??

Q--TEST FOR WHETHER OR NOT A FRISK FOR WEAPONS IS ALLOWED?????

APPLICATION OF THAT TEST TO THE FACTS HERE.????

JUSTIFICATION OF SEARCH IS TO LOOK FOR WEAPONS SO WHAT IS THE SCOPE???

SCOPE IS BASED ON PURPOSE.

WAS SCOPE OF SEARCH OF TERRY O.K????

SO------WHAT IS THE HOLDING???? (FAMOUS WORDS)

DOUGLAS—DISSENT—WHAT WAS HIS ARGUMENT

IT WAS A SEIZURE AND A SEARCH AND PROBABLE CAUSE REQUIRED.

Q—COULD OFFICER SEARCH FOR DRUGS?????

WHAT IF NO REASONABLE SUSPICION THAT TERRY ARMED OR DANGEROUS----COULD OFFICER FRISK????

REASONABLE SUSPICION FOR STOP DOES NOT EQUAL JUSTIFICATION FOR FRISK. MUST BE REASONABLE SUS. FOR STOP AND THEN REAS. SUSP. TO BELIEVE SUSPECT ARMED OR DANGEROUS FOR FRISK.

1LEVELS OF INTERACTION BETWEEN PERSON & LEO

—VOLUNTARY ENCOUNTER4th Amendment does not apply----no “seizure”No level of suspicion necessaryPerson is free to leave or terminate encounter

—SEIZURES (4TH AMENDMENT APPLIES)

----STOPPerson is not free to “leave”(to decline officer’s requests or terminate encounter)

37

Page 38: Crim Pro Outline

Reasonable suspicion is level of probability that is necessary to comply with 4th Amendment

—ARRESTPerson not free to leave & based on length, location & purpose of detention (“suspect”), detention is deemed an arrestProbable cause is level of probability necessary to comply with 4th Amendment

Differences between 3 (when voluntary encounter becomes seizure& when “stop” becomes “arrest” are subtle and fact sensitive.

Stops and arrests are seizures, thus invoking protection of 4th. Thus, evidence gained as a fruit of an unreasonable one is subject to suppression. Further, if arrest and not sufficient probable cause, any evidence gained in search incident to that arrest is inadmissible.

Pennsylvania v. MimmsWhen an officer legally stops a driver on the highway, he may order the driver or passenger out of the car without further justification because of the legitimate and weighty interest in police safety.

Dunaway v. New York, Florida v. Royer & United States v. Sharpe

All dealt with “drawing lines” between a “Terry seizure” (requiring only reasonable suspicion) and an arrest (requiring probable cause).

An investigatory detention must be temporary and last no longer than is necessary to effectuate the purpose of the stop. In assessing whether a detention is too long in duration to be justified as an investigatory stop, the court considers it appropriate to examine whether the police diligently pursued a means of investigation that was likely to confirm or dispel their suspicions quickly, during which time it was necessary to detain the defendant.

DRAWING LINES BETWEEN A “VOLUNTARY ENCOUNTER” (NON-SEIZURE) AND A SEIZURE

U.S. v. Mendenhall (1980)When woman who fit a drug courier profile was stopped and strip searched and told to just give them the drugs, the court held that she had consented and that this was a voluntary encounter. A person is seized only when, by means of physical force or a show of authority, his freedom of movement is restrained. A person has been seized within the meaning of the Fourth Amendment only if, in view of the circumstances surrounding the incident, a reasonable person would have believed that he was not free to leave. Mendenhall was in a voluntary encounter and was free to leave.

38

Page 39: Crim Pro Outline

U.S. v. Drayton (2002)Christopher Drayton and Clifton Brown were traveling on a Greyhound bus. In Tallahassee, Florida, police officers boarded the bus as part of a routine interdiction effort. One of the officers worked his way from back to front, speaking with individual passengers as he went. The officer did not inform the passengers of their right to refuse to cooperate. As the officer approached Drayton and Brown, he identified himself, declared that the police were looking for drugs and weapons, and asked if the two had any bags. Subsequently, the officer asked Brown whether he minded if he checked his person. Brown agreed and a pat-down revealed hard objects similar to drug packages in both thigh areas. When Drayton agreed, a pat-down revealed similar objects. Both were arrested. A further search revealed that Drayton and Brown had taped cocaine to their legs. Charged with federal drug crimes, Drayton and Brown moved to suppress the cocaine on the ground that their consent to the pat-down searches was invalid. In denying the motions, the District Court determined that the police conduct was not coercive and Drayton and Brown's consent to the search was voluntary. In reversing, the Court of Appeals noted that bus passengers do not feel free to disregard officers' requests to search absent some positive indication that consent may be refused. Must police officers, while searching buses at random to ask questions and to request passengers' consent to searches, advise passengers of their right not to cooperate? No. In a 6-3 opinion delivered by Justice Anthony M. Kennedy, the Court held that the Fourth Amendment does not require police officers to advise bus passengers of their right not to cooperate and to refuse consent to searches. The Court reasoned that, although the officer did not inform the defendants of their right to refuse the search, he did request permission to search and gave no indication consent was required. Moreover, the Court noted, the totality of the circumstances indicated that the consent was voluntary. Justice David H. Souter, with whom Justices John Paul Stevens and Ruth Bader Ginsburg joined, dissented. "The issue we took to review is whether the police's examination of the bus passengers ... amounted to a suspicionless seizure under the Fourth Amendment. If it did, any consent to search was plainly invalid as a product of the illegal seizure," argued Justice Souter.

California v. Hodari (p. 391)Hodari D's defense relies upon the proposition that a seizure occurs "when the officer, by means of physical force, or show of authority, has in some way restrained the liberty of a citizen." The language of the 4th amendment cannot sustain his contention. The word "seizure" readily bears the meaning of lying on of hands or application of physical force to restrain movement, even when it is ultimately unsuccessful. It does not remotely apply to a police officer telling a fleeing subject to "stop in the name of the law." That is not a seizure. An arrest requires either physical force or, where that is absent, submission to the assertion of authority. Assuming that Officer Pertoso's pursuit constituted a "show of authority" enjoining Hodari D to halt, since Hodari D didn't comply with that injunction he was not seized until he was tackled. The cocaine abandoned while he was running was not the fruit of an illegal seizure. Principle of law: A seizure must have a physical component to it. Simply running away does not constitute a seizure of one's liberty

Brendlin v. California (2007)A person is seized by the police and thus entitled to challenge the government’s action under the Fourth Amendment when the officer by means of physical force or show of authority terminates or restrains his freedom of movement, through means intentionally applied. There is no seizure without actual submission.

39

Page 40: Crim Pro Outline

Court summarizes “seizure of person” cases

Based on such decides that not only the driver but also the car passenger is “seized” when police order a traffic stop

FACTORS THAT CAN INFLUENCE WHETHER VOLUNTARY ENCOUNTER OR SEIZURE

# of officersIn uniform?Weapons? Drawn?Threats by officersPhysical touching or any application of forceTone of voiceLocationBlocking of doors or exitsTaking ticket, drivers’ license or I.D.Telling person not free to leave

TERRY ESTABLISHED NEW LEVEL OF PROBABILITY THAT JUSTIFIES A “STOP” (SEIZURE)

REASONABLE SUSPICION

QUESTION---How much or what quality of evidence needed to satisfy the reasonable suspicion threshold??

Alabama v. White (1990)Defendant appeals from a conviction of possession of marijuana and cocaine, after a consensual search of her car revealed drugs. Defendant claims that there was not sufficient indicia of reliability to provide reasonable suspicion to make the initial investigatory stop. "An anonymous telephone tip, as corroborated by independent police work is sufficient indicia of reliability to provide reasonable suspicion to make an investigatory stop of defendant's vehicle." Prediction of future behavior demonstrates insider information. Leaving in station wagon full of cocaine and anonymous tip. In Terry stop, reasonable suspicsion can arise from information that is less reliable than probable cause

As we have seen before, Defendant’s theory is based on fruit of poisonous tree---she consented to the search but her argument is that the consent is the fruit of the unreasonable stop (the poisonous tree)

Florida v. J.L. (Note 4 on page 402)

Court held there was not reasonable suspicion because they just state a fact (young black men, one had a gun right now) and no prediction of future acts.How is Florida v. J.L. distinguishable from Adams v. Williams (Note 3, p. 401) where court

40

Page 41: Crim Pro Outline

found evidence sufficient for reasonable suspicion? If informant has given correct information about other things before, this may be enough to create reasonable suspicion.

Note 7—p. 405

Note 5, p. 402

IN LIGHT OF WHREN, WHICH ALLOWS PRETEXTUAL STOPS AND TERRY’S LOWER STANDARD OF PROBABILITY, RACIAL PROFILING HAS BECOME A SERIOUS ISSUE…..INFORMATION ON P. 403 ILLUSTRATES THAT THE PROBLEM IS NOT SPECULATIVE

WHAT ABOUT NEW LAW IN ARIZONA ALLOWING POLICE TO STOP IF REASONABLE SUSPICION TO BELIEVE THAT PERSON IN COUNTRY ILLEGALLY AND ALLOWING INQUIRY RE: STATUS?

Hiibel case---Note 6, p. 404

Nevada had statute requiring those justifiably (reasonable suspicion) stopped by police to identify themselves. Hibel was arrested pursuant to “resisting and obstructing” statute because he failed to provide identification (after being asked 11 times) when stopped for public intoxication.

Supreme Court applied reasonableness balancing in determining whether “stop and identify” statutes constitutional. Only constitution if there is reasonable suspicion for a Terry st

Illinois v. Wardlow (p. 406)Sudden flight in a high crime area is enough for a Terry stop. Presence alone in a high crime area is not enough.

Notice that Court draws distinction between ignoring police and going about business (which it stated in Royer was o.k.) and unprovoked flight

Problem 7B—p. 405-406

Reasonable suspicion?

Problem—Note 3, p. 411

PROTECTIVE SWEEPS

MARYLAND V. BUIE (p. 411)Officers at Buie’s home with arrest warrantLooked in basement after Buie emerged from thereFound a red running suit that was connected to the robberyIs running suit admissible? Could they search basement?

41

Page 42: Crim Pro Outline

BROADER ISSUE?

The Fourth Amendment permits a properly limited protective sweep in conjunction with an in-home arrest when the searching officer possesses a reasonable belief based on specific and articulable facts that the area to be swept harbors an individual posing a danger to those on the arrest scene. The real issue in this case is the plain view doctrine of the Fourth Amendment. Specifically, the plain view doctrine allows a police officer to seize contraband or evidence in plain view without first obtaining a warrant provided that (1) he or she was lawfully located in the area where the evidence was observed and (2) the incriminating nature of the evidence was immediately apparent. Because the description of the bank robber included a red running suit, the "immediately apparent" prong was satisfied. The only question was whether the officer that made the observation was lawfully in the basement when he made the observation. Court held that: As an incident to the arrest officers may, as a precautionary matter and without probable cause or reasonable suspicion, look in closets and other spaces immediately adjoining the place of arrest from which an attack could be immediately launched. This portion of the holding has recently been called into question.[2]If an officer wishes to conduct a sweep of the premises, there must be articulable facts which, taken together with the rational inferences from those facts, would warrant a reasonably prudent officer in believing that the area to be swept harbors an individual posing a danger to those on the arrest scene.

Michigan v. Long (1983)If, while conducting a legitimate Terry search of the interior of the automobile, the officer should, as here, discover contraband other than weapons, he clearly cannot be required to ignore the contraband, and the Fourth Amendment does not require its suppression in such circumstances.

FROM BUIEWHEN MAKING AN ARREST IN AN ENCLOSED AREA (HOME OR BUILDING), OFFICER CAN, AS A PRECAUTIONARY MATTER, LOOK IN CLOSETS AND OTHER SPACES IMMEDIATELY ADJOINING PLACE OF ARREST---PLACES FROM WHICH AN ATTACK COULD BE IMMEDIATELY LAUNCHED. THIS REQUIRES NO PROBABLE CAUSE OR REASONABLE SUSPICION.BEYOND THOSE AREAS (I.E.---THE BASEMENT IN BUIE), MUST BE ARTICULABLE BELIEF OR SUSPICION THAT SOMEONE IS THERE WHO POSES A DANGER.IN BOTH OF THE ABOVE CASES, THE SEARCH OR PROTECTIVE SWEEP MAY ONLY BE A CURSORY INSPECTION OF THOSE AREAS WHERE A PERSON MAY BE FOUND (I.E.---OFFICER CAN’T LOOK IN CONTAINERS, DRAWERS, ETC.) SEARCH IS LIMITED TO THE PURPOSE AND SCOPE.

MAJORITY OF FEDERAL CIRCUITS (NOT ALL) HOLD THAT PROTECTIVE SWEEP DOCTRINE IS NOT LIMITED TO THE CONTEXT OF AN ARREST AND ALSO APPLIES IF THE OFFICER IS LEGALLY IN HOME OR OTHER LOCATION FOR LEGITIMATE LAW ENFORCEMENT PURPOSES

OFFICER SAFETY RATIONALE HAS BEEN EXTENDED TO ALLOW “FRISKS” OR “PROTECTIVE SWEEPS” OF CARS AS WELL (SEE MICHIGAN V. LONG, pp.419-420 OF BOOK)----IF, AFTER VALID STOP OF CAR, OFFICER HAS ARTICULABLE BELIEF

42

Page 43: Crim Pro Outline

THAT SUSPECT DANGEROUS AND COULD GRAB A WEAPON FROM THE CAR, OFFICER MAY SEARCH PASSENGER COMPARTMENT OF CAR FOR WEAPONS (CAN ONLY LOOK PLACES WHERE WEAPON COULD BE) THIS RESULTS IN YET ANOTHER REASON WHY A CAR SEARCH WITHOUT A WARRANT CAN BE FOUND TO BE REASONABLE.

1EXCEPTIONS TO THE EXCLUSIONARY RULE

1. IMPEACHMENT OF DEFENDANT

Exception does not apply to involuntary statements that violate due processnor, according to majority of courts, to true violations of 6th A.

–Must relate to D’s testimony on direct or question asked on cross (and questions asked on cross must be reasonably related to testimony on direct)

-- Cannot use to impeach the testimony of a witness other than Defendant (James v. Illinois—p. 476)

2. PROCEEDINGS THAT ARE NOT CRIMINAL TRIALS

-examples include civil trials, administrative proceedings including deportation proceedings; parole revocation hearings; sentencing hearings and grand jury proceedings; one exception-----exclusionary rule does apply to civil forfeiture

proceedings.

3. GOOD FAITH DOCTRINE

a. Only comes into play if there is a warrant involved that later is found to be invalid so first question to ask is----Did police rely on invalid warrant??

b. Since purpose of the exception is that the exclusionary rule should not be applied to deter reasonable law enforcement activity, the exception only applies where the police officer’s reliance on the warrant was objectively reasonaable.

c. Test-----Would a reasonably, well-trained officer have known that the warrant was bad despite the magistrate’s authorization

d. EXCEPTIONS???(Situations where good faith doctrine cannot be relied upon and the exclusionary rule will apply)

4. BASED ON HUDSON V. MICHIGAN (p. 509) EXCLUSIONARY RULE DOES NOT APPLY TO VIOLATIONS OF KNOCK AND ANNOUNCE RULE

43

Page 44: Crim Pro Outline

U.S. V. LEON (p. 477)

The exclusionary rule requires that evidence illegally seized must be excluded from criminal trials. Leon was the target of police surveillance based on an anonymous informant's tip. The police applied to a judge for a search warrant of Leon's home based on the evidence from their surveillance. A judge issued the warrant and the police recovered large quantities of illegal drugs. Leon was indicted for violating federal drug laws. A judge concluded that the affadavit for the search warrant was insufficient; it did not establish the probable cause necessary to issue the warrant. Thus, the evidence obtained under the warrant could not be introduced at Leon's trial. Is there a "good faith" exception to the exclusionary rule? Yes, there is such an exception. The justices held that evidence seized on the basis of a mistakenly issued search warrant could be introduced at trial. The exclusionary rule, argued the majority, is not a right but a remedy justified by its ability to deter illegal police conduct. In Leon, the costs of the exclusionary rule outweighed the benefits. The exclusionary rule is costly to society: Guilty defendants go unpunished and people lose respect for the law. The benefits of the exclusionary rule are uncertain: The rule cannot deter police in a case like Leon, where they act in good faith on a warrant issued by a judge.

COURT’S MODE OF ANALYIS----BALANCING

???????? VS. ??????????

ACCORDING TO COURT, WHAT MUST AN INVALID WARRANT DO IN ORDER TO HAVE A DETERRENT EFFECT?

ACCORDING TO HOLDING, WHAT ARE FOUR INSTANCES WHEN OFFICER CANNOT RELY ON GOOD FAITH EXCEPTION TO EXCLUSIONARY RULE

1. If the magistrate or judge in issuing a warrant was misled by information in an affidavit that the affiant knew was false or would have known was false except for his reckless disregard of the truth, or

2. if the issuing magistrate wholly abandoned his detached and neutral judicial role. 3. Nor would an officer manifest objective good faith in relying on a warrant based on an

affidavit so lacking in indicia of probable cause as to render official belief in its existence entirely unreasonable.

4. Finally, depending on the circumstances of the particular case, a warrant may be so facially deficient -- i.e., in failing to particularize the place to be searched or the things to be seized -- that the executing officers cannot reasonably presume it to be valid.

MASS V. SHEPPARD (1984) GROH V. RAMIREZ (2004)P. 490 P. 491

44

Page 45: Crim Pro Outline

Affidavit established Affidavit established probable causeprobable cause

Warrant application listedPreprinted warrant form particulars but actual warrantdidn’t meet particularity failed particularity requirement&requirement failed to incorporated supporting

application by reference

Magistrate–“I’ll makechanges.”—made some butnot all.

Sheppard: Officer used the wrong form and authorized him to search for narcotics but the judge told him he had made the proper changes. Court held the officer shouldn’t have to doubt the judge so the good faith exception was met.

Groh: The warrant failed to describe the objects to be seized and therefore failed the particularity requirement. The warrant was invalid.

HERRING V. U.S. , 129 S.Ct. 695 (2009)—p. 525

Herring went to Coffee County Sheriff’s office to check on impounded truck Sheriff suspected he might have arrest warrant When sheriff didn’t find a warrant he called sheriff’s office in neighboring Dale County----the warrant clerk for that sheriff’s office said that there was an active warrant As Herring drove away in his truck, Coffee County sheriff stopped himand arrested him Incident to the arrest, Herring’s truck is searched and meth is found.Meanwhile, Dale County sherrif’s clerk finds out that the warrant had actually been recalled—there was no outstanding arrest warrant. The court held that as long as the officers are acting in good-faith and that their conduct was not deliberate, reckless or grossly negligent, than the exclusionary rule does not apply.

TEST—WHETHER COSTS OF EXCLUDING EVIDENCE ARE OUTWEIGHED BY DETERRENT EFFECT---“TO TRIGGER THE EXCLUSIONARY RULE, POLICE CONDUCT MUST BE SUFFICIENTLY DELIBERATE THAT EXCLUSION CAN MEANINGFULLY DETER IT AND SUFFICIENTLY CULPABLE THAT SUCH DETERRENCE IS WOTH THE PRICE PAID BY THE JUSTICE SYSTEM

1EXCLUSIONARY RULE

APPLIES TO EVIDENCE THAT IS DIRECT PRODUCT OF VIOLATION ITSELF (THE POISONOUS TREE)

ALSO APPLIES TO EVIDENCE DERIVED FROM THE ILLEGALLY OBTAINED

45

Page 46: Crim Pro Outline

EVIDENCE—THE FRUIT OF THE POISONOUS TREE

EXAMPLE

Police search trunk pursuant to search incident to arrest

Violation of 4th???

Find gun and notebook in trunk. Notebook contains information on where other guns are located. Use information from notebook to get warrant to search building. Find other guns in building.

Gun and notebook are direct product of violation (poisonous tree)—excluded

Other guns found in building are “fruit of poisonous tree.”

EXAMPLES FROM CASES WE’VE ALREADY LOOKED AT

Pringle----

Claimed that arrest violated 4th because not based on probable cause

Attempting to get “confession” suppressed claiming it was “fruit” of the illegal arrest (the poisonous tree)

Court said arrest o.k. because probable cause. (NO POISONOUSTREE)

Drayton (bus case)

Claimed that he was illegally “seized” on the bus (no reasonable suspicion)

Attempting to get evidence gained from a search to which he consented claiming that his consent was “fruit” of the illegal seizure.

Court said not seized. (NO POISONOUS TREE)

LIMITATIONS ON FRUIT OF POISONOUS TREE DOCTRINE

1. INDEPENDENT SOURCE---- Evidence is admissible if it is discovered by means wholly independent of any constitutional violation—if the “fruit” is found

46

Page 47: Crim Pro Outline

by legal means unrelated to the original “poisonous tree conduct”—admissible

Example–Officers conduct illegal search of car (the poisonous tree) and find list of names & addresses; Obtain warrant to search one of addresses using info from search;

Simultaneously, other officers involved in independent investigation conduct an undercover buy at same address; undercover officer believes cover blown, fears for safety and signals back-up officers to enter house

Information discovered in this search (legal search based on probable cause and exigent circumstances) admissible even though officers with “tainted” warrant may discover same info.

Legal search was independent from illegal car search

Murray v. U.S.–p. 496

Facts: Based on information received from informants, federal law enforcement agents had been surveilling petitioner Murray and several of his co-conspirators. At about 1:45 p.m. on April 6th, 1983, they observed Murray drive a truck and Carter drive a green camper, into a warehouse in South Boston. When the petitioners drove the vehicles out about 20 minutes later, the surveilling agents saw within the warehouse two individuals and a tractor-trailer rig bearing a long, dark container. Murray and Carter later turned over the truck and camper to other drivers who were in turn followed and ultimately arrested, and the vehicles lawfully seized. Both vehicles were found to contain marijuana. After receiving this information, several of the agents converged on the South Boston warehouse and forced entry. They found the warehouse unoccupied, but observed in plain view numerous burlap-wrapped bales that were later found to contain marijuana. They left without disturbing the bales, kept the warehouse under surveillance, and did not reenter it until they had a search warrant. In applying for the warrant, the agents did not mention the prior entry, and did not rely on any observations made during that entry. When the warrant was issued at 10:40p.m., approximately eight hours after the initial entry, the agents immediately reentered the warehouse and seized 270 bales of marijuana and notebooks listing customers for whom the bales were destined. Procedural Posture: Petitioners moved to suppress the evidence found in the warehouse. District Court: Denied the motion, rejecting the petitioner’s argument that the warrant was invalid because the agents did not inform the Magistrate about their prior warrantless entry, and that the warrant was tainted by that entry. Court of Appeals: Affirmed

Issue(s): Whether the independent source doctrine applies if evidence is initially discovered unlawfully, but is later obtained lawfully in a manner independent of the original discovery?

Judgment/Disposition: Vacated and remanded

Holding: The court held that the independent doctrine does apply if the evidence was seen during an unlawful search but is later obtained during a lawful search, as long as the evidence in question was not the basis for probable cause to issue the warrant.

47

Page 48: Crim Pro Outline

Majority: Writing for the majority, Justice Scalia rejected the petitioner’s argument that the independent source doctrine only applies to evidence obtained for the first time during an independent lawful search. Instead, he embraces the government’s argument that this doctrine also applies to evidence initially discovered during, or as consequence of, an unlawful search, but later obtained independently from activities untainted by the initial illegality. Scalia also rejected the argument that this would create more incentives for police to conduct illegal searches, than obtain a warrant. He wrote “We see the incentives differently. An officer with probable cause sufficient to obtain a search warrant would be foolish to enter the premise first in an unlawful manner. By doing so, he would risk suppression of all the evidence on the premises, both seen and unseen, since his action would add to the normal burden of convincing a magistrate that there is probable cause the much more onerous burden of convincing a trial court that no information gained from the illegal entry affected either the law enforcement officers decision to seek a warrant or the magistrates decision to grant it. Nor would the officer without sufficient probable cause to obtain a search warrant have any added incentive to conduct an unlawful entry, since whatever he finds cannot be used to establish probable cause before a magistrate”.

DOES THE INDEPENDENT SOURCE DOCTRINE PROVIDE POLICE WITH INCENTIVES TO DISREGARD CONSTITUTION? 2. INEVITABLE DISCOVERY( very similar to independent source)

Tainted evidence (fruit of poisonous tree) admissible if State proves by preponderance of evidence that the evidence would have been inevitably discovered by lawful means, independent of illegal activity. Additionally–State must show that the means would have actually been used.

Example—Police have warrant to search apartment for Defendant; officer finds notebook in drawer; opens and reads; notes indicate that there are illegal guns in the closet; opens closet and finds sawed-off shotgun. State would argue inevitable discovery because gun would have been inevitably discovered in search for D.

Nix v. Williams

48

Page 49: Crim Pro Outline

Williams arrested and arraigned in Davenport, Iowa in connection with disappearance of 10 year old girl from the YMCA in Des Moines Iowa.

Police surmised that the body was located somewhere near the interstate from Des Moines to Davenport and began a search.

In the meantime, Williams was driven back to Des Moines by detectives.

Despite being advised by Williams’ attorneys not to question him, detectives give himTheir “Christian burial speech.” (actual speech on p. 705 of your book)

Reacting to speech, Williams agrees to direct them to the body.

Search had been called off for day but body only 2 & ½ miles from where called off.

Motion to suppress all evidence of body as “fruit of poisonous tree”

What was poisonous tree????

In previous Supreme court case (which we will consider in a few weeks), Supreme Court held that Williams’ statement was inadmissible because it was “deliberately elicited”and thus violated his 6th Amendment right to counsel.

WHAT WAS HOLDING OF SUPREME COURT ABOUT EVIDENCE CONCERNING THE BODY?

REASONING?? Reason for exclusionary rule???

Goal and proper balancing between interest in deterring police misconduct and having juries receive all evidence is reached when put police (state) in SAME position they would have been in had violation not occurred.

Q---But—if exclude evidence that comes from independent source what position is state put in?????

RULE:If govt can prove that evidence would have been obtained inevitably and thus admitted regardless of any overreaching by police, no reason to keep from jury.

So–then–Court must examine whether body here would have inevitably been discovered.

DECISION??

49

Page 50: Crim Pro Outline

ISSUE HAS ARISEN AS TO WHETHER INEVITABLE DISCOVERY PRINCIPLE APPLIES TO PRIMARY AS WELL AS SECONDARY EVIDENCE

Example—Trunk of car searched incident to arrest (violation of 4th)Police find drugs and a drug ledger. Info in drug ledger leads to other physical evidence. Drugs and drug ledger are direct or “primary” evidence. “Other physical evidence” is indirect or “secondary” evidence.

Evidence would have inevitably been discovered in an inventory search of car. (as long as policy & policy followed)

MAJORITY OF COURTS HAVE HELD THAT INEVITABLE DISCOVERY RULE APPLIES TO BOTH TYPES OF EVIDENCE.

3. ATTENUATION OF TAINT

The initial “taint”(the poisonous tree) has been attenuated by a subsequent occurrence....something else, sufficiently distinguishable, happens which purges the original “taint”

Example—Defendant illegally arrested but released and then, 2 weeks later,voluntarily goes to the police station and confesses. Statement admissible because the “poisonous” tree (the “taint”) has been attenuated by D’s voluntary confession.

WONG-SUN

Mr. Toy arrested based on reasonable suspicion

Made statement implicating Yee

Yee arrested and searched—drugs found in bedroom

Can drugs that were found be used against Toy??

Why or why not?

Yee made statement implicating Toy and Wong Sun

Wong Sun arrested and released

Several days later, Wong Sun voluntarily went to police station and gave statement.

50

Page 51: Crim Pro Outline

Is Wong Sun’s statement fruit of the “poisonous” arrest made without probable cause?

According to Court, what is the test?

Excluding the presentation of verbal evidence and recovered narcotics where they were both fruits of an illegal entry. Narcotics agents unlawfully entered Toy's laundry at which point Toy indicated that Yee was selling narcotics. The drug agents then went to Yee and found the narcotics. Yee made a deal to give up his supplier, Wong Sun. Wong Sun was a prominent businessman, so the police invited him for a conversation about the case. Following this conversation, Wong Sun voluntarily returned to the police station to make a deal of his own, during the process of which he confessed. At Yee's trial, Toy's statements and the discovered drugs were both excluded as fruit of the poisonous tree because the search was done without a warrant. Wong Sun's lawyer argued that Wong Sun's confession should also be excluded as fruit of the poisonous tree. The court affirmed the fruit of the poisonous tree rule, but found an exception to exclusion in Wong Sun's case on the grounds that Wong Sun had voluntarily returned to the police station to make his statement, an act which "attenuated" or broke the chain of evidence. Wong Sun was granted a new trial, but his confession was admissible.

Brown v. Illinois (Note 2, -p. 507)

FACTS

Police broke into Brown’s apartment and searched it.

Waited for him to arrive home and arrested him based only on suspicion–no warrant and no probable cause

Twenty minute drive to police station; in car, questioned him about the murder

Put him in room a police station and spread murder file out in front of him

Mirandized—gives statement ; ASA comes in–more questioning

Mirandized again-----Gives confession.

POISONOUS TREE??? FRUIT??

Did Miranda warnings attenuate the taint??

FACTORS APPLIED BY COURT:

51

Page 52: Crim Pro Outline

1. The length of time that has elapsed between the initial illegality and the seizure of the fruit in question;

2. The flagrancy of the initial misconduct (dissipation of bad faith violations takes longer than with good-faith violations);

3. The existence ir absence of intervening causes of the seizure of the fruit; and4. The presence or absence of an act of free will by the defendant resulting in the seizure of

the fruit.

Kaupp v. Texas, 538 U.S. 626

Similar

Police picked up 19 year old at 3 A.M., handcuffed him, said “we need to go talk” and took him, shoeless and still in boxer shorts to the police station. They stopped for 10 minutes at the scene of a murder that they “suspected” he had been involved in. At police station they Mirandized him and after 15 minutes, he admitted having a part in the crime.

Poisonous tree??

Fruit??

While Court remanded case to trial court to decide, Court expressed serious doubts that the “taint” had been attenuated. Court noted the short amount of time that had passed between illegal arrest and statement, the fact that officers knew they didn’t have probable cause to arrest and his having to remain in a partially clothed state. Pointed out that Miranda warnings do not necessarily break the connection between the illegality and the statement and noted that all other factors point the other way.

WHAT COURT GIVETH, COURT CAN TAKE AWAY

Exclusionary rule is not part of 4th Amendment but instead aCourt-created remedy for constitutional violations.

Relying on such, (and other convoluted reasoning) the court in 2006 in Hudson v. Michigan Ruled that the exclusionary rule does not apply to evidence gained in violation of the “knock and announce” rule.According to Court, interests violated have nothing to do with seizure of evidence and thus there is no need to apply exclusionary rule.Also-----Seem to conclude that no need for deterrence in this circumstance---mention police professionalism and fact that constitutional rights of citizens are taken seriously

Court mentions other remedies that Defendants have (civil suits, etc.)

PROBLEM

52

Page 53: Crim Pro Outline

Exterminators discover locked closet in Darstraum’s apartment

Manager lets them in to closet

Exterminators discover marijuana and growing operation

Manager calls police

Office Polgers arrives

Manager lets Polgers into apartment and closet

Polgers gets everyone out and calls narcs

Narcs come and interview everyone

Prepare affidavit and gets search warrant

Affidavit includes Polgers observations but detective said she would haveattempted to get warrant without them

Search of apartment pursuant to warrant and seizure of stuff.

Darstraum files Suppression Motion. GROUNDS???How should judge rule??

1CONFESSIONS/STATEMENTS

SUBJECT TO EXCLUSION UNDER FOUR THEORIES

1.FRUIT OF POISONOUS 4TH AMENDMENT VIOLATION (Example---Brown v. Illinois—no probable cause for arrest & statement obtained while “arrested”)

2. MUST COMPLY WITH DUE PROCESS----STEMS FROM DUE PROCESS CLAUSES OF 5TH AND 14TH AMENDMENTS

STATEMENT MUST BE VOLUNTARYAPPLIES EVEN IF NO CUSTODY OR INTERROGATION

3. MUST COMPLY WITH MIRANDA—STEMS FROM 5TH

AMENDMENT RIGHT TO BE FREE FROM SELF-INCRIMINATION

STATEMENT CANNOT BE COMPELLED AND IN ORDER NOT TO BE COMPELLED, COMPLIANCE WITH MIRANDA

53

Page 54: Crim Pro Outline

NECESSARY

MIRANDA ONLY APPLIES IF THERE IS CUSTODIAL INTERROGATION

4. STATEMENT MUST NOT BE TAKEN IN VIOLATION OF 6TH

AMENDMENT RIGHT TO COUNSEL

NO DELIBERATE ELICITATION IF RIGHT TO COUNSEL HAS ATTACHED

VOLUNTARINESS REQUIREMENT

State has burden of showing that, under totality of circumstances, Defendant’s statement is a product of Defendant’s own free will

2 prong TEST to refute voluntariness

1. Did a government actor (police) subject Defendant to coercion?

2. Was the coercion sufficient to overcome the will of Defendant??

Factors to considerObjective----conduct of policeSubjective—particular vulnerability of suspect

A. Use of force or fearB. Length of InterrogationC. Deprivation of Bodily NeedsD. Use of psychological techniques—especially pressure tacticsE. Promises of LeniencyF. DeceptionG. Characteristics of Accused-----age; mental capacity; emotional state; level of intelligence

and education

Ramifications

If judge (in some states–jury) finds statement involuntary, statement cannot be used for any purpose (including impeachment)Any “fruit” of involuntary statement also excludedIf Defendant convicted and finding of “voluntariness” reversed on appeal, conviction reversed unless “harmless” error.

54

Page 55: Crim Pro Outline

Brown v. Mississippi (1934)

Raymond Stuart, a white planter, was murdered on March 30, 1934. Arthur Ellington, Ed Brown and Henry Shields, three black tenant farmers, were arrested for his murder. At the trial, the prosecution's principal evidence was the defendants' confessions to police officers. During the trial, however, prosecution witnesses freely admitted that the defendants confessed only after being subjected to brutal whippings by the officers. One defendant had also been subjected to being strung up by his neck from a tree in addition to the whippings. Torture was then used in order to extract confessions from the defendants. The confessions were nevertheless admitted into evidence. This was the only evidence used in the subsequent one-day trial. The defendants were convicted by a jury and sentenced to be hanged; and the convictions were affirmed by the Mississippi Supreme Court on appeal.

In a unanimous decision, the Court reversed the convictions of the defendants. The opinion was delivered by Chief Justice Hughes. It was decided that a defendant's confessions that is extracted by police violence cannot be entered as evidence and violates the Due Process Clause of the Fourteenth Amendment.

The Fifth Amendment guarantees the defendant's protection from self incrimination, such as through torture as applied in this case. The Fourteenth Amendment's Due Process clause was used to apply this provision of the Fifth Amendment to the states. This was one case in a series of cases in which parts of the Bill of Rights have been deemed "fundamental" enough to apply to the states as well as in federal cases.

Colorado v. Connelly (p. 554)Schizophrenic confessed to murder. Connelly significantly changed the voluntariness standard - the test used to determine the admissibility of confessions under the due process clauses of the Fifth and Fourteenth Amendments.[3] Before Connelly the test was whether the confession was voluntary considering the totality of the circumstances.[4] "Voluntary" carried its everyday meaning: the confession had to be a product of the exercise of the defendant's free will rather than police coercion.[5] After Connelly the totality of circumstances test is not even triggered unless the defendant can show coercive police conduct.[6] Questions of free will and rational decision making are irrelevant to a due process claim unless police misconduct existed and a causal connection can be shown between the misconduct and the confession.

Spano v. New York (p. 555)

Spano was an immigrant in his mid-twenties with a junior high school education. He shot a person after a bar fight. He fled the crime scene and was indicted for murder while he was in hiding. Spano called Gaspar Bruno, a close friend of his who training to become a police officer. Spano told Bruno that the deceased had injured him, and that he intended to get a lawyer and turn himself in to law enforcement. Bruno relayed the information to his superiors. Spano, along with his newly appointed attorney, turned himself in the day following his conversation with Bruno.

55

Page 56: Crim Pro Outline

Spano was questioned continuously for several hours and was told he could not consult with his attorney. The police provided him with dinner during his first night of questioning. The following day, Spano was transferred to another police station where questioning continued. He was again denied assistance of counsel. Bruno, upon police instructions, told Spano that he could get into trouble if Spano did not confess although Bruno's job was not really in jeopardy. Bruno approached Spano four times before Spano gave a statement; each time questioning had resumed, Spano requested assistance of counsel. Police escorted Spano to the location where they believed he had disposed of the murder weapon. While searching for the weapon, Spano confessed.

The issue was whether police violated Spano's Sixth Amendment right to counsel during interrogation. The Court did not reach the Sixth Amendment question, however, because they held that the use of the confession was inconsistent with the Fourteenth Amendment and fundamental fairness. The Court identified six factors that together constituted police misconduct:

1. Spano was relatively young and inexperienced in the criminal justice system.2. Spano was subjected to leading questions and did not make a narrative statement to

police.3. He was questioned incessantly and through the night.4. Police persisted questioning him even though he said his attorney advised him to remain

silent.5. Police ignored his request to contact his attorney.6. The officers used his close friend, Bruno, to manipulate him.

The Court held that the interrogation violated Spano's 14th Amendment due process rights because Spano's confession was not voluntary.

The two concurring opinions emphasized Spano's right to counsel.

Spano opened the door for Miranda v. Arizona. Even though the majority opinion used the traditional voluntariness analysis, the concurring opinions indicated that a person had constitutional a right to counsel, if that counsel had been retained, once the person is formally charged by indictment or information. The majority opinion did not preclude the right to counsel argument expressed in the concurring opinions.

Arizona v. Fulimante

While Fulimante in prison for one offense, he became friends with Sarivola, another inmate who was a paid FBI informant. Sarivola heard a rumor that Fulimante a suspect in a child’s murder. S raised issue with F but F repeatedly denied involvement.

clarifying the standard of review of a criminal defendant's allegedly coerced confession.

56

Page 57: Crim Pro Outline

In 1982, the 11-year-old stepdaughter of one Oreste Fulminante was murdered in Arizona. Later, Fulminante was incarcerated for an unrelated crime. While in prison, Fulminante met Anthony Sarivola, a fellow inmate, who was also a confidential informant for the Federal Bureau of Investigation. Sarivola offered Fulminante protection from "tough treatment" in prison in exchange for a confession to the murder of Fulminante's stepdaughter. Fulminante agreed, confessing to Sarivola that he murdered his stepdaughter. As a result, Fulminante was charged with the murder, and his confession to Sarivola was used against him at trial.

The trial court denied Fulminante's motion to suppress the confession on the basis that it was coerced because Fulminante might have been subject to violence in prison had he not confessed. Fulminante was convicted and sentenced to death, and appealed to the Arizona Supreme Court, which held that the confession was indeed coerced. Reasoning that a harmless error analysis was inappropriate in the case of involuntary confessions, the court ordered a new trial.

In a divided opinion, the United States Supreme Court held that the state supreme court's finding that the threat of violence aimed at Fulminante was credible enough to establish a finding of coercion, and therefore affirmed the reversal. In addition, the Court held that a harmless error analysis should nonetheless be applied to any allegedly coerced confession. In either case, the Court held that a new trial was warranted.

TEST (according to Court)?

Court’s holding and reasoning therefore?

PROBLEM

Marvoal, single woman lives with 3 young children

11:30 at night

3 federal agents and federal prosecutor come to home; say they need to speak to her about investigation of her boss

Sit in small room and demand co-operation

Threaten her with prosecution if she doesn’t co-operate

Say they’ll file a forfeiture action and she’ll lose her house.

She’s afraid—no relatives to care for kids.

Hears kid cry and agent prevents her from leaving room.

Says she’ll cooperate and feels like she can then leave.

57

Page 58: Crim Pro Outline

When she returns, makes statement.

If prosecuted and you represent her, what motion should you make and what are grounds?? Arguments??

State’s response??

Likely ruling??

EXCLUSION OF STATEMENT PURSUANT TO MIRANDA IS BASED ON 5TH AMENDMENT PROVISION

NO PERSON SHALL BE COMPELLED IN ANY CRIMINAL CASE TO BE A WITNESS AGAINST HIMSELF………

AT TRIAL CANNOT BE FORCED TO TESTIFY

AND

MIRANDA PROVIDES PROTECTION FOR CERTAIN PRE-TRIAL INCRIMINATION

APPLIES TO ANYONE IN U.S. OR SUBJECT TO INTERROGATION BY U.S. AGENTS OUTSIDE U.S.

VIOLATION OCCURS WHEN STATEMENT INTRODUCED AT TRIAL

ONLY APPIES TO EVIDENCE THAT IS TESTIMONIAL IN NATURE

Examples of types of non-testimonial evidence that is not protected By 5th Amendment include fingerprints, blood samples, handwritingexemplars, voice identification, lineup identification, etc.

MIRANDA CHECKLIST

1. TEST FOR APPLICABILITY

A. SUSPECT IN CUSTODY

B. INTERROGATION BY PERSON WHO IS KNOWN TO BE GOVERNMENT ACTOR

2. IF APPLICABLE----WERE WARNINGS ADEQUATE?

58

Page 59: Crim Pro Outline

3. WAS THERE A KNOWING, INTELLIGENT AND VOLUNTARY (KIV) WAIVER OF MIRANDA RIGHTS?

If so, and warnings adequate, no Miranda violation

4. WAS THERE AN ADEQUATE INVOCATION OF RIGHTS AND, IF SO, WAS IT SCRUPULOUSLY HONORED?

5. CONSEQUENCES OF VIOLATION OF MIRANDA

6. EXCEPTIONS TO NECESSITY OF COMPLIANCE WITH MIRANDA

MIRANDA1 DECISION (p. 581)

TO WHAT WAS THE DECISION A RESPONSE???

WHAT REASONS DOES COURT ASSERT FOR 5TH AMENDMENT PRIVILEGE???

ACCORDING TO COURT, NEED PROPER SAFEGUARDS TOCOMBAT INHERENTLY COMPELLING PRESSURE.....SO–WHEN ARE SAFEGUARDS NECESSARY???

DOES IT APPLY TO ON-SCENE INVESTIGATIVEQUESTIONING??

DOES IT MATTER IF SUSPECT AWARE OF RIGHTS???

IF MIRANDA APPLIES, WHAT MUST POLICE DO???

WHAT IS COURT’S RATIONALE FOR NEED FOR ATTORNEY?

WHAT MUST POLICE DO IF I SAY I DON’T WANT TO TALK??

59

Page 60: Crim Pro Outline

WHAT MUST HAPPEN IF I SAY I WANT TO TALK TO MY ATTORNEY??

EFFECT OF NON-COMPLIANCE WITH MIRANDA??

The Court was called upon to consider the constitutionality of a number of instances, ruled on jointly, in which defendants were questioned "while in custody or otherwise deprived of [their] freedom in any significant way." In Vignera v. New York, the petitioner was questioned by police, made oral admissions, and signed an inculpatory statement all without being notified of his right to counsel. Similarly, in Westover v. United States, the petitioner was arrested by the FBI, interrogated, and made to sign statements without being notified of his right to counsel. Lastly, in California v. Stewart, local police held and interrogated the defendant for five days without notification of his right to counsel. In all these cases, suspects were questioned by police officers, detectives, or prosecuting attorneys in rooms that cut them off from the outside world. In none of the cases were suspects given warnings of their rights at the outset of their interrogation.

Question: 

Does the police practice of interrogating individuals without notifiying them of their right to counsel and their protection against self-incrimination violate the Fifth Amendment?

Conclusion: 

The Court held that prosecutors could not use statements stemming from custodial interrogation of defendants unless they demonstrated the use of procedural safeguards "effective to secure the privilege against self-incrimination." The Court noted that "the modern practice of in-custody interrogation is psychologically rather than physically oriented" and that "the blood of the accused is not the only hallmark of an unconstitutional inquisition." The Court specifically outlined the necessary aspects of police warnings to suspects, including warnings of the right to remain silent and the right to have counsel present during interrogations.

DISSENT’S ARGUMENTS???CUSTODY

Oregon v. Mathiason, p. 645

An Oregon state police officer suspected Carl Mathiason of burglary and asked him to come to the police station for questioning. Mathiason came freely, spoke with the officer, and was not arrested at the time. He was arrested later and a trial court used evidence obtained during the questioning to convict him. Mathiason moved to suppress the evidence since he was not read his Miranda rights before the questioning. The court admitted the evidence since Mathiason was not in custody during the questioning. The Oregon Court of Appeals affirmed. The Supreme Court of Oregon reversed since it found that Matianson was in a "coercive environment" when questioned and therefore deserved to hear his Miranda rights.

60

Page 61: Crim Pro Outline

Question: 

Can incriminating evidence obtained from a suspect during a voluntary interview be used if the police did not read Miranda rights to the suspect?

Conclusion: 

Yes. In a per curiam decision, the Court held that its decision in Miranda v. Arizona only required law enforcement officials to recite a suspect's rights when suspect had been "deprived of his freedom of action in any significant way." The Court determined that in this case there was "no indication that the questioning took place in a context where respondent's freedom to depart was restricted in any way." Even if the police coercively pressured Mathiason during the interview, he came to the police station freely and was free to leave at any time. Therefore Miranda rights did not apply.

Berkemer v. McCarty (p. 648)

1. Miranda applies to custodial interrogations involving minor traffic offenses.

2. Routine questioning of motorists detained pursuant to traffic stops is not custodial interrogation under Miranda.

The Miranda rule prohibits the use of testimonial evidence in criminal proceedings that is the product of custodial police interrogation unless the police properly advised the defendant of his Fifth Amendment rights and the defendant knowingly, intelligently and voluntarily waived those rights and agreed to talk to the police. The circumstances triggering the Miranda safeguards are "custody" and "interrogation." Custody means formal arrest or the deprivation of freedom to an extent associated with formal arrest. Interrogation means explicit questioning or actions that are reasonably likely to elicit an incriminating response. Unquestionably the defendant in Berkemer was interrogated. In fact, he was interrogated twice - prearrest roadside questionig and post arrest questioning at the jail. In neither case had the officer advised the defendant of his Miranda rights. As for post arrest interrogation, the defendant was in custody since he had been arrested. The issue for the court was whether to create an exception to Miranda for custodial interrogations that related to minor offenses.[15] The Supreme Court declined to carve out such an exception because to do so would sacrifice the certainty and clarity of the Miranda rule.[16] The pre arrest interrogation raised the issue of whether detention was equivalent to custody for purposes of the Miranda rule. [17]The court found that there were two significant differences between interrogation of person taken into custody and detainees. First was the length of the detention. Investigative detentions were brief and usually culminated in the issuance of a citation and release of the defendant. Second, the circumstances attendant to roadside detention were substantially less coercive and compulsive than those typically surrounding custodial interrogation. [18]Specifically the Court noted that during most traffic stops the actions of the officer were "exposed" to public view and that stops typically involved only one or two officers.[19]

61

Page 62: Crim Pro Outline

A police officer can stop a vehicle if he has a reasonable articulable reason to suspect that “criminal activity is occurring.” [20]

The officer may detain for sufficient time to conduct a reasonable investigation that either confirms or dispels his suspicions

The officer is not required to arrest the suspect once the officer has PC. The officer may delay the arrest for purposes of conducting a non-custodial interrogation.

Officer may interrogate the suspect without advising him of his Miranda rights

The officer may ignore suspect's attempts to exercise her Miranda righs because the right have not attached.

For example, if a person who is being detained on suspicions of impaired driving asks to contact a lawyer the officer may ignore the statement and continue to question the suspect.

HOLDING?PROBLEM 6 (A)---p. 655

Is response admissible??

Why or why not?

PROBLEM 6 (B)

Are answers admissible??

Under Mathiason test??

Under Berkemer test??

PROBLEM 6 (D)—p. 656

Custody??

Why or why not??

INTERROGATION

62

Page 63: Crim Pro Outline

Rhode Island v. Innis (p. 656)

Innis suspected of robbery and murder of cab driverPicked up—given Miranda warningsHe said he wanted to talk to lawyer—invoked right to lawyer; questioning is supposed to stop.Put in “wagon” to be driven to police station

Then what happened???

Issue?

According to Court, does Miranda apply only to actual “questioning” by officers??

What is test for interrogation??

Application of test to facts and holding of Court?

After a picture identification by the victim of a robbery, Thomas J. Innis was arrested by police in Providence, Rhode Island. Innis was unarmed when arrested. Innis was advised of his Miranda rights and subsequently requested to speak with a lawyer. While escorting Innis to the station in a police car, three officers began discussing the shotgun involved in the robbery. One of the officers commented that there was a school for handicapped children in the area and that if one of the students found the weapon he might injure himself. Innis then interrupted and told the officers to turn the car around so he could show them where the gun was located.

Question: 

Did the police "interrogation" en route to the station violate Innis's Miranda rights?

Conclusion: 

No. In a 6-to-3 decision, the Court held that the Miranda safeguards came into play "whenever a person in custody is subjected to either express questioning or its functional equivalent," noting that the term "interrogation" under Miranda included "any words or actions on the part of the police (other than those normally attendant to arrest and custody) that the police should know are reasonably likely to elicit an incriminating response from the subject." The Court then found that the officers' conversation did not qualify as words or actions that they should have known were reasonably likely to elicit such a response from Innis.

Problem 4 (A) p. 663

63

Page 64: Crim Pro Outline

Interrogation?

Problem 4 (C), p. 664

In custody??

Interrogation?

NOTE 6 (p. 664-665)

Pennsylvania v. MunizRespondent/Defendant:- Muniz; the defendant was arrested for driving while drunk and he was taken to the police station. At the police station, the officer asked Muniz for his name, address, height, weight, eye color, date of birth, and current age. Then the officer asked Muniz for the date of his 6th birthday, which Muniz could not remember. Then he made Muniz to perform three sobriety tests and the defendant failed at every test. All this was videotaped and Muniz was not read his Miranda rights before he was asked to perform all the tests. Muniz was read his Miranda rights after he refused to take the breathhalyzer test. The defendat argues that the taperecording of all the proceedings that took place at the police station should be suppressed because it is self incriminating evidence which was obtained before he was read his Miranda Rights and that violated his 5th Amendment rights. The Court of Appeals suppressed the evidence and remanded the case, and now the state appeals.

Issue: Should the entire audio portion of the videotape be suppressed?

Holding: No

Key Facts: Custodial interrogation according to Rhode Island v. Innis is "any words or actions on the part of the police (other than those normally attendant to arrest and custody) that the police should know are reasonable likely to elicit an incriminating response from the suspect."

Legal Reasoning: The court reasoned that 5th Amendment and Miranda apply only to evidence of testimonial or communicative nature and not to "real or physical evidence." The court ruled that when the officer asked the defendant for his name, height, etc. before reading him his Miranda rights, it did not violate the 5th Amendment rights of the defendant because these questions are part of police booking routine. The court further ruled that the defendant's drunk style of speech was part of the physical evidence which is not protected under Miranda. The court also ruled that taperecording of defendant performing the sobriety tests and the defendant

64

Page 65: Crim Pro Outline

refusing the breathhalyzer test do not violate his 5th Amendment rights because these transactions can not be considered custodial interrogation. The court reasoned that the officers only asked Muniz to perform the tests and they did not ask Muniz any questions which are "reasonably likely to elicit an incriminating response from the suspect." At last, the court ruled that the question about Muniz's sixth birthday should be suppressed because it can be considered testimonial evidence which should have been obtained after Muniz was read his Miranda rights.

1WAIVER

LIKE OTHER CONSTITUTIONAL RIGHTS, THE 5TH AMENDMENT RIGHT TO BE FREE FROM SELF-INCRIMINATION CAN BE WAIVED.

IF THERE HAS BEEN A VALID WAIVER OF MIRANDA RIGHTS, THEN ANY STATEMENT MADE IS ADMISSIBLE.

THE STATE HAS THE BURDEN OF PROVING, BY A PREPONDERANCE OF THE EVIDENCE, THAT THERE IS A VALID WAIVER.

WHAT IS TEST FOR VALID WAIVER???Must be voluntary, knowing and intelligent.

North Carolina v. Butler, p. 666

The question of waiver must be determined on the particular facts and circumstances surrounding that case, including the background, experience, and conduct of the accused.

Note 4, p. 670

Suspect arrested for one crime, waived Miranda rights and questioned aboutDifferent crime

MIRANDA RIGHTS ARE “CUSTODY” SPECIFIC, NOT “OFFENSE OR CHARGE” SPECIFIC. YOU WILL SEE, HOWEVER THAT 6TH AMENDMENT RIGHT TO COUNSEL IS “OFFENSE OR CHARGE” SPECIFIC, NOT “CUSTODY” SPECIFIC.

Moran v. Burbine (Note 5, p. 670)

Suspect’s lawyer called and said she would act as lawyer during any questioning. Police told her—no interrogation tonight Suspect waived Miranda rights---not told about lawyer. Suspect knowingly and voluntarily waived right to counsel---doesn’t matter that counsel was trying to reach him.

65

Page 66: Crim Pro Outline

Notes 6 and 7—p. 671

Thoughts???

Arguments you would make if prosecutor or defense counsel??

TO AVOID SOME OF THESE PROBLEMS, SOME JURISDICTIONS REQUIRE VIDEOTAPING OF AT LEAST SOME INTERROGATIONS

Berghuis v. Thompkins (2010)

Thompkins given form---were warnings on form adequate?

Police had T read 5th (extra) warning outloud to make sure he could read and concluded heUnderstood English

T refused to sign form—conflicting evidence about whether he indicated that he Understood his rights.

Interrogation started

Pertinent info about what happened while being questioned???“Do you pray to God to forgive you for shooting that boy down?”

T---Yes

Motion to Suppress statement based on violation of Miranda denied by trial court

A Michigan state court convicted Van Chester Thompkins of first-degree murder, assault with intent to commit murder, and several firearms related charges. After exhausting his remedies in Michigan state court, Thompkins petitioned for habeas corpus relief in a Michigan federal district court. The district court denied the petition.

On appeal, Thompkins argued that his confession was obtained in violation of the Fifth Amendment and that he was denied effective counsel at trial. The Sixth Circuit held that the Michigan Supreme Court's finding that Thompkins waived his Fifth Amendment right was unreasonable because Thompkins refused to sign an acknowledgement that he had been informed of his Miranda rights and rarely made eye contact with the officer throughout the three hour interview. The Sixth Circuit also held that the Michigan Supreme Court improperly determined that Thompkins was not prejudiced by his counsel's failure to request a limiting instruction related to his separately tried co-defendant's testimony.

66

Page 67: Crim Pro Outline

Question: 

1) Did the Sixth Circuit improperly expand the Miranda rule when it held that defendant's Fifth Amendment rights were violated?

2) Did the Sixth Circuit fail to give the state court deference when it granted habeas corpus relief with respect to defendant's ineffective counsel argument when there was substantial evidence of the defendant's guilt?

Conclusion: 

Yes. Yes. The Supreme Court reversed the Sixth Circuit, holding that the state court's decision to reject Mr. Thompkins' Miranda claim was correct. With Justice Anthony M. Kennedy writing for the majority, the Court reasoned that Mr. Thompkins failed to invoke his Miranda rights to remain silent and to counsel because he failed to do so "unambiguously." Moreover, the Court reasoned that Mr. Thompkins waived his Miranda right to remain silent when he "knowingly and voluntarily" made a statement to the police. The Court further held that, even if Mr. Thompkins' counsel was ineffective, he cannot show he was prejudiced by counsel's deficient performance – a prerequisite to establishing that his Sixth Amendment right was violated.

Justice Sonia Sotamayor, joined by Justices John Paul Stevens, Ruth Bader Ginsburg, and Stephen G. Breyer, dissented. She reprimanded the majority for retreating from the broad protections afforded by Miranda, stating that now a criminal suspect waives his rights simply by uttering a "few one-word responses."

Test for valid waiver?

Must a waiver be express??

Test when no express waiver??

Court’s holding regarding waiver??

Three factors that informed majority opinion on the waiver issue??

FROM CASE

“In sum, a suspect who has received and understood the Miranda warnings and has not invoked his Miranda rights, waives the right to remain silent by making an uncoerced statement to the police. Thompkins did not invoke his right to remain silent and stop the questioning. Understanding his rights in full, he waived his right to remain silent by making a voluntary

67

Page 68: Crim Pro Outline

statement to the police. The police, moreover, were not required to obtain a waiver of Thompkins’ right to remain silent before interrogating him.”

“Today’s decision turns Miranda upside down. Criminal suspects must now unambiguously invoke their right to remain silent which, counterintuitively, requires them to speak. At the same time, suspects will be legally presumed to have waived their rights even if they have given no clear expression of their intent to do so. Those results, in my view, find no basis in Miranda or our subsequent cases and are inconsistent with the fair-trial principles on which those precedents are grounded.”

CONSEQUENCES OF INVOCATION OF MIRANDA RIGHTS

IF INVOKE RIGHT TO REMAIN SILENT

According to Miranda decision—“interrogation must cease”

Michigan v. Mosley (p. 678)

Mosley questioned by detective in armed robbery section. Questioning ended as soon as he said he “didn’t want to answer any questions about the robberies”

What happened a few hours later???

Same detective?

Same crime?

Did Mosley waive Miranda rights?

What does Court say the test is for admissibility of statements made afterperson has invoked right to remain silent??

Application of test to facts here and factors considered by Court?

Holding?

REMEMBER---- Must have subsequent KIV waiver

68

Page 69: Crim Pro Outline

Court held that a criminal suspect's assertion of his right to remain silent after a Miranda warning does not preclude the police from re-Mirandizing him and questioning him about a different crime.

Richard Bert Mosley was arrested in Michigan in connection with several robberies. Police Mirandized him and asked him questions about the robberies. He eventually asserted his right to remain silent and interrogation ceased. A few hours later, while Mosley was still being held, another officer interrogated him regarding a homicide. The officer read Mosley a fresh Miranda warning and Mosley then made statements that incriminated him. The Court held that the questioning on a different crime did not violate Mosley's right to remain silent. The Court reasoned that his Fifth Amendment rights were scrupulously honored when he ended questioning about the robberies. Given the significant time lapse and the fresh set of Miranda warnings, there was no violation of Mosley's Fifth Amendment rights.

IF INVOKE RIGHT TO COUNSEL

According to Miranda decision, what must happen if suspect requests lawyer??

Edwards v. Arizona, P. 674

What happened?

Decision and reasoning of Arizona Supreme Court?

Issue before Supreme Court?

HOLDING???

The court held that a waiver of the right to counsel, once invoked, not only must be voluntary, but also must constitute a knowing and intelligent relinquishment of a known right or privilege. The fact that Edwards confessed after being read his Miranda rights does not demonstrate that he understood right to counsel and intelligently and knowingly relinquished it. Once the right to counsel under the Fifth Amendment has been invoked, a valid waiver cannot be shown just by the accused responding to interrogations despite Miranda warning's having been read. In Rhode Island v. Innis, 446 U.S. 291 (1980), the court clarifies the meaning of interrogation.

[edit] Contrast to the Sixth Amendment right to counsel

Under Montejo v. Louisiana, the Sixth Amendment's right to counsel does not prevent police from initiation of interrogation once the right attaches. This is in contrast to the Fifth Amendment right to counsel, where the police custodial interrogation is not permitted as per Edwards.

69

Page 70: Crim Pro Outline

What if suspect initiates further communication??

Note 2A—p. 679

Suspect invoked right to counsel and later asked “well, what is going to happen to me now?”

According to Court in Oregon v. Bradshaw,( p. 679), what is test for “initiation” of further communication??

To decide this case, the Court had to determine whether the Oregon Court of Appeals had correctly applied Edwards v. Arizona, 451 U.S. 477 (1981). In Edwards, the Court had held that "an accused, having expressed his desire to deal with the police only through counsel, is not subject to further interrogation by the authorities until counsel has been made available to him, unless the accused himself initiates further communication, exchanges, or conversations with the police." The question presented in this case was the effect of Bradshaw's query, "Well, what is going to happen to me now?"

The Oregon Court of Appeals had concluded that Bradshaw had waived his right to counsel when he asked what would happen to him. This, the Court concluded, was not how the Edwards rule should be applied. Edwards was meant to protect an accused from being badgered by the police. Hence, if the accused himself approaches the police, this suggests that the accused may not be being badgered, so that when the police respond to the accused's queries with questions of their own, a reviewing court can decide, based on the totality of the circumstances, whether the accused had waived his Fifth Amendment rights during interrogation. In other words, an accused does not waive his right to counsel during interrogation merely because, after invoking that right, he approaches the police with questions of his own.

Bradshaw's question here, "Well, what is going to happen to me now?", "initiated" conversation with the police "in the ordinary dictionary sense of the word." It "evinced a willingness and a desire for a generalized discussion about the investigation; it was not merely a necessary inquiry arising out of the incidents of the custodial relationship." Thus, a suspect detained during an investigation may, after invoking his right to have counsel present during an interrogation, may ask for a drink of water or to use the telephone without retreating from his prior request for counsel. He may not, however, ask more generalized and open-ended questions. Accordingly, the Court held that the Edwards rule had not been violated. The police did not otherwise make any threats, promises, or inducements to Bradshaw in order to persuade him to talk; in fact they confirmed that he had the right to counsel before resuming their questioning. Under these circumstances, Bradshaw had voluntarily waived his right to counsel during interrogation.

70

Page 71: Crim Pro Outline

Was there “initiation” here?

Note 2C—p. 680

Admissible?

Minnick v. Mississippi ( p. 680)

Minnick invoked right to counsel and consulted with lawyer on 2 or 3 occasions. Police from different jurisdiction came and questioned him—no waiver.

COURT----Statements inadmissible…once a suspect invokes right to counsel, police may not reinitiate interrogation in the absence of counsel.

Maryland v. Shatzer (2010 case)---2 & ½ years elapsed between Edwards invocation and 2nd waiver request. Had been released from custody in interim. Court said waiver valid and subsequent statements admissible. In dicta----after 2 weeks, renewed interrogation o.k.(this “cleansing period” gives suspect time to consult with friends, counsel, etc. and “shake-off” any residual effects of prior custody)

Davis v. U.S. (p. 681)

Suspect initially waived rights

1 & ½ hours later----“Maybe I should talk to a lawyer.”

Discussion and then “No, I’m not asking for a lawyer; No I don’t want a lawyer.”

Then more questioning.

Issue---Did Davis invoke his right to counsel by above statements??

TEST??Suspect must unambiguously request counsel.

HOLDING?

Problem 6 (D)---p.684

71

Page 72: Crim Pro Outline

Admissible??

Problem 7—p. 684-685

ARE MIRANDA WARNINGS CONSTITUTIONALLY REQUIRED???

Policy created by Court; not part of Constitution

Whether or not Miranda warnings constitutionally required was inquestion for a # of years preceding Dickerson

18 U.S.C.3501 (1968) making voluntariness the sole test for theadmissibility of statements in federal court.

Some lower federal courts had indicated that some confessions takenin violation of Miranda were not violative of Constitution.

Oregon v. Elstad (1985)—Supremes said same and called Miranda a“prophylactic” rule

Then—2000—Dickerson v. U.S.

Justice Dept disagreed with 3501 and had not used it for 20 years.

U.S. appealed a trial court decision suppressing a statement thatviolated Miranda but did not raise 3501

Conservative organization filed amicus brief raising it.

4th Cir reversed trial court–invoked statute and indicated that Miranda compliancenot required under Constitution.

Supreme Court granted cert and held that Miranda announced a constitutional rule that Congress could not supersede legislatively and, following rule of stare decisis, Supreme Court not willing to overrule Miranda.

The Court issued the above edict despite its decision 15years earlier in Oregon v. Elstad.

And, since Dickerson, Court has said that because Miranda is prophylactic and not part of “core” 5th Amendment rights, the only remedy for violation is exclusion of statement at trial.

72

Page 73: Crim Pro Outline

Oregon v. Elstad (p. 616)

Elstad a suspect in a burglary. Police go to his home with a warrant for his arrest.Ask him to get dressed and accompany them into living room. Sit down with him and ask him questions. No Miranda warnings.

He says “Yes, I was there.”

Miranda violation?

Then what happened?

Issue?

Holding of Oregon Supreme Court?

Holding of U.S. SUPREMES??

Reasoning?

Michael James Elstad was suspected of committing a burglary and was picked up by police officers in his home. Before officers had given the warnings required by Miranda v. Arizona, Elstad made an incriminating statement. Once at the Sheriff's headquarters, Elstad was advised of his rights. Elstad then voluntarily executed a written confession.

Question: 

Was Elstad's written confession made invalid by the failure of the officers to administer Miranda warnings at his home?

Conclusion: 

In a 6-to-3 decision, the Court held that while Miranda required that unwarned admissions must be suppressed, subsequent statements, if made knowingly and voluntarily, need not be. The Court held that ". . .the mere fact that a suspect has made an unwarned admission does not warrant a presumption of compulsion." The Court also noted that police officers were ill-equipped to determine when "custody" legally begins. Justice O'Connor, writing for the majority, argued that the holding "in no way retreat[ed] from the bright-line rule of Miranda."

73

Page 74: Crim Pro Outline

Note 4—p. 622

But----Missouri v. Seibert (p. 631) left small opening

Unwarned questioning

Incriminating statement

Miranda warnings

Questioning covering same ground as unwarned questioning

A second incriminating statement

ISSUE BEFORE SUPREME COURT?

TEST??

FACTORS TO CONSIDER (according to Court)

HOLDING

Patrice Seibert was convicted of second degree murder for the death of 17-year-old Donald Rector, who died in a fire set in the mobile home where he lived with Seibert. Several days after the fire, Seibert was interogated by a police officer. The officer initially withheld her Miranda warnings, hoping to get a confession from her first. Once she had confessed, the officer took a short break from questioning, then read her her Miranda rights and resumed questioning her after she waived those rights. He prompted her to restate the confession that she had made earlier. Based on this second, Mirandized confession, Seibert was convicted.

She appealed, charging that the officer's intentional use of an un-Mirandized interrogation to get the initial confession made the later confession, though it occurred after she had waived her Miranda rights, inadmissable. The prosecution cited Oregon v. Elstad to argue that an initial, un-Mirandized confession did not make a defendant incapable of voluntarily waiving her Miranda rights and confessing later.

The Supreme Court of Missouri agreed with Seibert, overturning the conviction.

74

Page 75: Crim Pro Outline

Question: 

Does the rule from Oregon v. Elstad that a defendant who has made an un-Mirandized confession may later waive her Miranda rights to make a second confession (admissible in court) still apply when the initial confession is the result of an intentional decision by a police officer to withhold her Miranda warnings?

Conclusion: 

No. In a decision with no majority, a four-justice plurality found that the post-Miranda confession is only admissible - even if the two-stage interview was unintentional, as it was in Elstad - if the Miranda warning and accompanying break are sufficient to give the suspect the reasonable belief that she has the right not to speak with the police. Justice Anthony Kennedy, in a concurring opinion that provided the fifth vote, found that evaluating the warning and accompanying break was only necessary if the police used the two-stage interrogation intentionally. Justice Kennedy wrote, "The admissibility of postwarning statements should continue to be governed by Elstad's principles unless the deliberate two-step strategy is employed. Then, the postwarning statements must be excluded unless curative measures are taken before they were made."

Elstead and Seibert both dealt with statements that were “fruit” of unwarned statement.

What about physical fruits??

United States v. Patane (P. 643)

Facts?

ISSUE??

HOLDING?

Samuel Patane was arrested at his home for calling his ex-girlfriend in violation of a restraining order. During the arrest, police offers began reading Patane his Miranda rights. Patane told the officers that he knew his rights. The officers then stopped reading them, at which point Patane told police that he had a gun in his house. They searched the house with his permission and found the gun. As an ex-felon, Patane was not permitted to possess a gun and was prosecuted for possession.

During the trial on gun possession charges, Patane argued that his arrest violated the Fourth Amendment prohibition of unreasonable searches and seizures and the Fifth Amendment right not to incriminate oneself because there was not probable cause to arrest him and because the gun had been found as a result of an un-Mirandized confession.

75

Page 76: Crim Pro Outline

The district court initially ruled that there was not probable cause for his arrest and that it was therefore unconstitutional. A 10th Circuit Court of Appeals panel disagreed, holding that Patane's ex-girlfriend had given police probable cause for the arrest. However, the panel held that gun could not be used as evidence because it had been found as the result of an un-Mirandized (and therefore unconstitutional) confession. The government appealed, arguing that physical evidence found as the result of un-Mirandized testimony could be used in court, despite the fact that the testimony itself was inadmissable.

Question: 

Can physical evidence found as a result of un-Mirandized but voluntary testimony be used in court?

Conclusion: 

Yes. In a decision without a majority opinion, three justices wrote that the Miranda warnings were merely intended to prevent violations of the Constitution, and that because Patane's un-Mirandized testimony was not admitted at trial the Constitution (specifically the Fifth Amendment's protection against self-incrimination) had not been violated. Physical evidence obtained from un-Mirandized statements, as long as those statement were not forced by police, were constitutionally admissible. Two other justices also held that the physical evidence was constitutionally admissible, but did so with the understanding that the Miranda warnings must be accommodated to other objectives of the criminal justice system. They did not discuss whether the Miranda warnings were, in themselves, constitutionally required.

EXCEPTIONS TO MIRANDA

1. STATEMENTS OBTAINED IN VIOLATION OF MIRANDACAN BE USED TO IMPEACH DEFENDANT’S TESTIMONY

(Harris v. New York, p. 605)

2. PUBLIC SAFETY

New York v. Quarles (p. 607)

What statement & evidence did Quarles wish to exclude? (pre-Elstead)

What were circumstances surrounding statement?

Did Miranda apply? Why or why not?

76

Page 77: Crim Pro Outline

ISSUE?

HOLDING OF SUPREME COURT?

REASONING?

POINTS MADE BY DISSENTERS?

Note 3---p. 614----

Does Miranda bar admission of Mr. B’s answer to the question??

Note 5B (Problem)-p. 615

Is statement admissible?

Note 5D (Problem)-p. 615

Is the statement admissible?

Note 5E (Problem)—p. 615

Is C’s statement admissible?

1Miranda based on 5th Amendment right not to be compelled to incriminate oneself----applies to custodial interrogation.

An incriminating statement may also violate a person’s 6th Amendment right to counsel

Sixth Amendment-----guarantees counsel in all criminal prosecutions

Right to appointed counsel for indigents—application to States

1932---Powell v. Alabama-----right to appointed counsel in capital cases

1942—Betts v. Brady---Due process right to appointed counsel at stateExpense if failure to appoint counsel would be “offensive to the common and fundamental ideas of fairness.”

1963-----Gideon v. Wainwright—states must provide counsel to indigents in criminal cases. “The right of one charged with

77

Page 78: Crim Pro Outline

crime to counsel may not be deemed fundamental and essential to fair trials in some countries, but it is in ours.” (was deemed to

mean felony cases)

Application to misdemeanors

Argersinger v. Hamlin and Scott v. Illinois (1972 and 1979)

In both state and federal systems, counsel must be provided in any caseinvolving a “term of imprisonment”

When entitled to counsel???

Sixth Amendment right to counsel extends to every “critical stage” of the case.

Does Defendant have 6th Amend right to counsel during pre-trial interrogation by the State?

MASSIAH case p. 696

held that the Sixth Amendment to the United States Constitution prohibits the government from eliciting statements from the defendant about themselves after the point that the Sixth Amendment right to counsel attaches.

In Massiah, the defendant had been indicted on a federal narcotics charge. He retained a lawyer, pled not guilty, and was released on bail. A co-defendant, after deciding to cooperate with the government, invited Massiah to sit in his car and discuss the crime he was indicted on, during which the government listened in via a radio transmitter. During the conversation, Massiah made several incriminating statements, and those statements were introduced at trial to be used against him.

Massiah appealed his conviction, which was affirmed in part by the Court of Appeals for the Second Circuit, 307 F.2d 62. The Supreme Court granted certiorari and reversed, holding that the statements made by the defendant outside the presence of his attorney must be suppressed.

The Massiah rule applies to the use of testimonial evidence in criminal proceedings deliberately elicited by the police from a defendant after formal charges have been filed. The events that trigger the Sixth Amendment safeguards under Massiah are (1) the commencement of adversarial criminal proceedings and (2) deliberate elicitation of information from the defendant by governmental agents.

The Sixth Amendment guarantees a defendant a right to counsel in all criminal prosecutions. The purposes of the Sixth Amendment right to counsel are to protect a defendant’s right to a fair trial and to assure that our adversarial system of justice functions properly by providing competent

78

Page 79: Crim Pro Outline

counsel as an advocate for the defendant in his contest against the “prosecutorial forces” of the state.

The Sixth Amendment right “attaches” once the government has committed itself to the prosecution of the case by the initiation of adversarial judicial proceedings "by way of formal charge, preliminary hearing, indictment, information or arraignment,".[2] Determining whether a particular event or proceeding constitutes the commencement of adversarial criminal proceedings requires both an examination of the rules of criminal procedure for the jurisdiction in which the crime is charged and the Supreme Courts cases dealing with the issue of when formal prosecution begins.[3] Once adversarial criminal proceedings commence the right to counsel applies to all critical stages of the prosecution and investigation. A critical stage is "any stage of the prosecution, formal or informal, in court or out, where counsel's absence might derogate from the accused's right to a fair trial." [4]

Government attempts to obtain incriminating statement related to the offense charged from the defendant by overt interrogation or surreptitious means is a critical stage and any information thus obtained is subject to suppression unless the government can show that an attorney was present or the defendant knowingly, voluntarily and intelligently waived his right to counsel.[5]

Deliberate elicitation is defined as the intentional creation of circumstances by government agents that are likely to produce incriminating information from the defendant.[6] Clearly express questioning (interrogation) would qualify but the concept also extends to surreptitious attempts to acquire information from the defendant through the use of undercover agents or paid informants.[7]

The definition of "deliberate elicitation" is not the same as the definition of "interrogation" under the Miranda rule. Miranda interrogation includes express questioning and any actions or statements that an officer would reasonably foresee as likely to cause an incriminating response. Massiah applies to express questioning and any attempt to deliberately and intentionally obtain incriminating information from the defendant regarding the crime charged. The difference is purposeful creation of an environment likely to produce incriminating information (Massiah) and action likely to induce an incriminating response even if that was not the officer's purpose or intent (Miranda).

The Sixth Amendment right to counsel is offense specific - the right only applies to post commencement attempts to obtain information relating to the crime charged.[8] The right does not extend to uncharged offenses even those that are factually related to the charged crime.[9]

As noted, information obtained in violation of the defendant's Sixth Amendment right to counsel is subject to suppression unless the government can establish that the defendant waived his right to counsel. The waiver must be knowing, intelligent and voluntary.[10] A valid Miranda waiver operates as a wavier of Sixth Amendment right.

79

Page 80: Crim Pro Outline

Contents[hide]

1 Massiah v. Miranda 2 Massiah and the Voluntariness Standard 3 See also 4 References

[edit] Massiah v. Miranda

1. Constitutional Basis - Miranda is based on the Fifth Amendment right to counsel and the Fifth Amendment right to remain silent. Massiah is based on the Sixth Amendment right to counsel.

2. Attachment - Miranda: Custody + Interrogation. (Charging status irrelevant) Massiah: Formally Charged + Deliberate Elicitation. (Custodial status irrelevant)

3. Scope a. Miranda applies to custodial interrogation by known governmental agents. Surreptitious acquisition of incriminating information allowed.

a. Massiah applies to overt and surreptitious interrogation.

b. Miranda is not offense specific.[11] b. Massiah is offense specific.[12]

c. Miranda: interrogation + "functional equivalent" c. Massiah: interrogation + "deliberate elicitation"

4. Waiver: Both Miranda and Massiah rights may be waived.

5. Assertion: In each case, the assertion must be clear and unequivocal. The effects of assertion are not identical. For purposes of Miranda, the police must immediately cease the interrogation and cannot resume interrogating the defendant about any offense charged or uncharged unless counsel is present or defendant initiates contact for purposes of resuming interrogation and valid waiver obtained. Because Massiah is offense-specific, an assertion of the sixth amendment right to counsel requires the police to cease interrogating the defendant about any charged offense. Apparently the police could continue questioning the defendant about uncharged crimes assuming that the defendant was not in custody. The defendant's remedy would be to leave or to refuse to answer questions.[13]

6. Remedy for violation: The remedy for violation of fifth and sixth amendment rights to counsel are identical. The statements and testimonial information is subject to suppression. Derivative evidence is not subject to suppression under Miranda - fruit of poisonous tree doctrine may apply to Massiah violation.[14] Both Miranda and Massiah defective statements can be used for impeachment purposes.

80

Page 81: Crim Pro Outline

7. Exceptions: The primary exceptions to Miranda are (1) the routine booking questions exception (2) the jail house informant exception and (3) the public safety exception. In Moulton v. Maine the Supreme Court refused to recognize a public safety exception to the Massiah rule.[15]

Massiah allows for the use of jail house informants provided the informants serve merely as "passive listeners."[16]

[edit] Massiah and the Voluntariness Standard

The Massiah rule is also to be contrasted with the vountariness standard of the Fifth and Fourteenth Amendments.[17] The voluntariness standard applies to all police interrogations regardless of the custodial status of the suspect and regardless of whether the suspect has been formally charged. The remedy for a violation of the standard is complete suppression of the statement and any evidence derived from the statement. The statement cannot be used as either substantive evidence of guilt or to impeach the defendant's testimony.[18] The reason for the strictness is the common law's aversion to the use of coerced confessions because of their inherent unreliability. Further the rights to be free from coerced confession cannot be waived nor is it necessary that the victim of coercive police conduct assert his right. In considering the voluntariness standard one must consider the Supreme Court's decision in Colorado v. Connelly.[19] Although federal courts' application of the Connelly rule has been inconsistent and state courts have often failed to appreciate the consequences of the case, Connelly clearly marked a significant change in the application of the voluntariness standard. Before Connelly the test was whether the confession was voluntary considering the totality of the circumstances.[20] "Voluntary" carried its everyday meaning - the confession had to be a product of the exercise of the defendant's free will rather than police coercion.[21] After Connelly the totality of circumstances test is not even trigered unless the defendant can show coercive police conduct.[22] Questions of free will and rational decision making are irrelevant to a due process claim unless police misconduct existed and a causal connection can be shown between the misconduct and the confession

Massiah and pal indicted for drug offenses

Massiah hired lawyer; released on bail

Massiah’s pal (Colson) agreed to co-operate with govt to get Massiah

Radio transmitter in Colson’s car

Colson asked Massiah to go for ride; initiated conversation

Massiah made incriminating statement

Does statement violate due process??

Pre-Miranda, but--Miranda violation?????

81

Page 82: Crim Pro Outline

Why or why not?

Massiah’s contention????

COURT’S HOLDING???

TWO PRONG TEST FOR 6TH PROTECTION

1. INITIATION OF ADVERSARIAL CRIMINAL PROCEEDINGSformal charge, prelim hearing, indictment, information or arraignment

2. DELIBERATE ELICITATION OF STATEMENT BY GOVT

Direct—asking questions, engaging D in conversation about case

Indirect

U.S. v. Henry 1980 (p. 700)

Informant/cellmate of Henry (Nichols) had been informant over a yearPaid if produced useful informationAgents knew he had access to Henry & conversing would not raise suspicionI/C told to “be alert to statements but don’t initiate conversation.”In conversation with Nichols, Henry made incriminating statements

Had Henry’s 6th Amendment rights attached??

Issue?

Holding and reasoning of Supreme Court?Court held that it was deliberate elicitation even if the agent’s statement that he did not intent that Nichols would take affirmative steps to secure incriminating information and that he must have known that such propinquity would lead to that result.

Kuhlmann v. Wilson 1986 (p. 701)

Informant/cellmate instructed to “keep ears open”No indication that he was paid for infoAsked no questions—only listened to spontaneous statements

82

Page 83: Crim Pro Outline

COURT HOLDING RE: DELIBERATE ELICITATION??

How distinguishable from Henry?

Reasoning of Court?Court held that since the sixth amendment is not violated whenever by luck or happenstance the State obtains incriminating statements from the accused after the right to counsel has attached, a defendant does not make a violation of that right simply by showing that an informant, either through prior arrangement or voluntarily, reported his incriminating statements to the police. Rather, the defendant must demonstrate that the police and their informant took some action, beyond merely listening that was designed deliberately to elicit incriminating remarks.

Maine v. Moulton—p. 782

Moulton and Colson indicted and assisted by counsel during arraignment

M and C meet to plan strategy----M proposes killing a witness

C then confessed to police and agrees to wear a bug and tape conversationswith M

M requested a meeting

C elicited incriminating statements during meeting and turned in tape.

Should statements be excluded based on violation of M’s 6th Amendment rights???

TEST??In Maine v. Moulton the court stated “By its very terms, it becomes applicable only when the government's role shifts from investigation to accusation. For it is only then that the assistance of one versed in the "intricacies . . . of law," ibid., is needed to assure that the prosecution's case encounters "the crucible of meaningful adversarial testing." The Sixth Amendment right to counsel does not attach until such time as the "government has committed itself to prosecute, and . . . the adverse positions of government and defendant have solidified ...'

COURT’S HOLDING??

WAIVER OF 6 TH AMENDMENT RIGHT

Brewer v. Williams p. 703

Already discussed facts when we looked at inevitable discovery doctrine

83

Page 84: Crim Pro Outline

Had Williams’ 6th Amendment rights attached?

Was there deliberate elicitation?

Iowa Supreme Court held that he had waived rights

Test for waiver of 6th?

Supreme Court holding on the waiver issue?

Three dissenting justices believed there was a valid waiver.

Note 6—p. 715 & 716

FRUIT OF POISONOUS TREE DOCTRINE APPLIES TO 6TH AMENDMENT VIOLATIONS----Here, however, in later case,Court held that “fruit” admissible because would have inevitably been discovered.

Michigan v. Jackson (Note 8, p. 716)

When arraigned, Jackson requested counsel. Although he later waived his Miranda right to counsel, Court held that this did not constitute a waiver of his 6th Amendment right to counsel. Court held since he had invoked his 6th Amendment right to counsel, he could not be questioned without counsel present.

Two years later came Patterson v. Illinois (p. 716)

Gang fight---Patterson involved in beating and drowning of rival gang member

Indicted for murder—significance?

While being transferred from one jail to another, asked a question and thenStarted to talk to officer about crime

Officer interrupted and gave him a Miranda card and read rights to him

Patterson initialed them and signed Miranda waiver form

84

Page 85: Crim Pro Outline

Gave lengthy incriminating statement

Same day—interview with ASA—again indicated he understood he was Waiving his rights and gave another statement

DEFENDANT’ ARGUMENT AS TO WHY STATEMENTS SHOULD BE SUPPRESSED?

CLAIM #1?

COURT HOLDING AND REASONING?

CLAIM #2?

TEST?

HOLDING AND REASONING?

ARGUMENTS OF DISSENTERS?

BEFORE Michigan v. Jackson OVERRULED BY SUPREMES INMontejo v. Louisiana (2009), TWO DIFFERENT SCENARIOS---

SCENARIO #1

IF NO WAIVER OF RIGHT, THEN STATEMENT WILL NOT BEADMISSIBLE

IF VALID WAIVER (KIV and which includes a Miranda waiver), STATEMENT ADMISSIBLE (Patterson v. Illinois)

SCENARIO #2

COUNSEL RETAINED, APPOINTED OR REQUESTED (INVOCATIONOF RIGHT)

STATE MAY ONLY APPROACH DEFENDANT THROUGH COUNSEL

85

Page 86: Crim Pro Outline

STATEMENT DELIBERATELY ELICITED WITHOUT DEFENDANTHAVING OPPORTUNITY TO CONSULT WITH LAWYER WILLNOT BE ADMISSIBLE (Even if Miranda warnings given and waiver obtained-----Michigan v. Jackson)

Montejo v. Louisiana (p. 727) changed this

EXPLICITLY OVERRULED Michigan v. Jackson

READ……..

RESULT---ONLY SCENARIO #1 ABOVE…..NO DISTINCTIONBETWEEN WHETHER OR NOT RIGHT TO COUNSEL IS INVOKED.

RIGHT CAN BE WAIVED, BASICALLY SAME WAY 5TH RIGHT TO COUNSEL IS WAIVED.

6TH AMENDMENT RIGHT IS OFFENSE SPECIFIC

McNeil v. Wisconsin—p. 727

McNeil arrested for robbery “A”

Brought before magistrate at bail hearing and had attorney appointed

Two days later, Mirandized, signed written waiver and questionedabout robbery “B”

Made incriminating statements and charged with robbery “B”

Moved to suppress claiming violated 6th Amendment right.

COURT’S HOLDING???

RATIONALE???

OFFENSE SPECIFIC

86

Page 87: Crim Pro Outline

Texas v. Cobb----p. 732 (not assigned)

D charged with burglary; appointed counsel

Police suspected he murdered burglary victims

(Factually related crimes)

Few months later----arrested and questioned about murder.

Confessed

Court held that statements made in response to questions about the murder were admissible.

“Offense” means charged offense plus any crimes that would be consideredsame offense for “double jeopardy” purposes. (Blockburger test)

IMPEACHMENT

STATEMENTS OBTAINED IN VIOLATION OF 6TH AMENDMENT CAN BE USED TO IMPEACH

Kansas v. Ventris (p. 735) (not assigned)

1EYEWITNESS IDENTIFICATION

FALSE IDENTIFICATION----BIGGEST CAUSE OF WRONGFULCONVICTIONS OVERALL

REASONS FOR EYEWITNESS IDENTIFICATIONS BEING UNRELIABLE???

(pp. 772-773)

THREE MAIN TYPES OF IDENTIFICATION PROCEDURES

1. LINEUPS

2. SHOWUPS

3. PHOTOSPREADS

ONE OF MAIN PROBLEMS WITH ALL IS POSSIBILITY OF SUGGESTION BY POLICE OFFICERS

87

Page 88: Crim Pro Outline

DUE TO PROBLEMS THAT CAN OCCUR, THERE ARE RULES BASED ON CONSTITUTIONAL RIGHTS THAT PROTECT INTEGRITY TO SOME EXTENT

1. 6th AMENDMENT RIGHT TO COUNSEL

2. DUE PROCESS PROTECTION FROM UNNECESSARILY SUGGESTIVE IDENTIFICATION

RIGHT TO COUNSEL----U.S. v. Wade, p. 761

9-21-64 Bank robbery

3-23-65 Wade indicted

4-26-65 Counsel appointed

May, 1965 LineupBoth employees identified WadeCounsel not present

Trial (some time later)Employees made in-court identification of Wade

Had Wade’s right to counsel attached????

Why???

On original appeal, attorneys argued lineup violated both 6th Amendment right to counsel and 5th Amendment right to be free from self-incrimination.

Was right to be free from self-incrimination violated????

Why or why not?

6 th AMENDMENT RIGHT TO COUNSEL

Accdg to Supremes,when is D entitled to counsel???

CRITICAL STAGE IF AFFECTS RIGHT TO FAIR TRIAL

88

Page 89: Crim Pro Outline

LINEUPS?

Why, according to Court in Wade???

HOLDING OF COURT RE: RESULTS OF LINEUP????

WHAT IF LINEUP HAD BEEN HELD PRIOR TO INDICTMENT???

CRITICAL STAGES???

SHOWUPS???

Moore v. Illinois, 434 U.S. 220 (1977)

Yes, but this particular “show-up” was at the preliminary hearing. Most show-ups occur prior to adversarial proceedings being initiated.

PHOTOSPREADS?

U.S. v. Ash—p.775

Does rule regarding right-to-counsel from Wade-Gilbert apply to photospreads?

Court pointed out that right is for defendant to have “assistance.” According to Court, since accused not present at these procedures, no possibility arises that will be misled by lack of familiarity with law or overpowered byProfessional adversary.

Court treats procedure as just part of discovery process---defense counsel canInterview witnesses, get copy of photos shown, etc.

WHAT DO YOU THINK???

BACK TO WADE—

Lineup results would not be admissible (actually, State did not try to introduce lineup results during trial in Wade, but in companion case, Gilbert v. California, lineup results excluded. (Note 1, p. 771)

89

Page 90: Crim Pro Outline

WHAT ABOUT COURTROOM IDENTIFICATION?

TEST, ACCORDING TO COURT?

WHO HAS AND WHAT IS EVIDENTIARY BURDEN?

FACTORS TO CONSIDER ???

IDENTIFICATION PROCEDURES CAN ALSO BE ATTACKED ON DUE PROCESS RIGHT OF 5TH & 14TH AMENDMENTS

Witness says robber had beard----Defendant only person with beard in lineup or photo array.

Victim is white woman; parents with her after rape. Alleged rapist, who is black is brought to scene in police car and handcuffed.

ANYTHING WRONG WITH ABOVE??

UNNECESSARY SUGGESTIVENESS

DUE PROCESS CLAUSE REQUIRES SUPPRESSION OF ANY OUT-OF COURT ID THAT, BASED ON TOTALITY OF CIRCUMSTANCES, WAS

1. UNNECESSARILY SUGGESTIVE

2. TO POINT THAT CREATES VERY SUBSTANTIAL LIKELIHOOD OF MISIDENTIFICATION

DEFENDANT HAS BURDEN Difficult to prove, generally

SINCE NOT BASED ON RIGHT TO COUNSEL, APPLIES TO PRE AND POST CHARGE ID PROCEDURES.

1ST Prong—some very suggestive situations have been found to be necessary

90

Page 91: Crim Pro Outline

Stovall v. Denno, p. 775

Court held that when man killed her husband and cops brought suspect to the hospital to view the suspect, handcuffed to the police, that this was not unnecessarily suggestive.

HOLDING OF COURT AND REASON THEREFORE?

2nd Prong —CREATING VERY SUBSTANTIAL LIKLIHOOD OF MISIDENTIFICATION

Manson v. Brathwaite, p.777Whether when the police officer was id’ing a suspect by being given only one photo and

positively id’ing him, that this would create a very substantial likelihood of misidentification. Court held that it was unnecessarily suggestive but that reliability is the lynch pin and based on the factors, this id did not create a very substantial likelihood of misidentification. (Officer was black, had a lot of time to look at him, gave a good description)

Factors include:The opportunity to viewThe degree of attentionThe accuracy of the descriptionThe witness’ level of certaintyThe time between the crime and the identification

IF OUT-OF-COURT VIOLATION OF DUE PROCESS, THEN IN-COURT ID SUPPRESSED IF PRIOR ID SO UNNECESSARILY SUGGESTIVE AS TO CREATE A VERY SUBSTANTIAL LIKELIHOOD OF IRREPARABLE MISIDENTIFICATION (VERY SIMILAR TO TEST WHEN VIOLATION OF 6TH AMENDMENT)

IF IDENTIFICATION EVIDENCE ADMITTED, RELIABILITY MAY BE ATTACKED BY DEFENSE COUNSEL AND ARGUED TO JURY.

PROBLEM—NOTE 2, P. 785PROBLEM

Man enters Brenda’s store, looks around for 3 minutes, leaves

20 minutes later, returns, buys beer, talks to her for 5 minutes, then pulls gun and robs her.

Later same day----Brenda views photo array—10 similar looking men.

91

Page 92: Crim Pro Outline

Defendant’s photo included but she picked photo of Manson

Said “I’m sure that’s the man.”

Lineup next day----Both Defendant and Manson included.

All in lineup similar build, coloring and appearance but Defendant 4 inches taller than anyone else.

Brenda ID’d Defendant and said “I’m 100% sure he’s the man. I know that the guy I picked out yesterday is in the line, but I was wrong. They look so different in person.”Should lineup identification be suppressed???

Should subsequent in-court identification be suppressed??

15 TH AMENDMENT (MIRANDA) 6 TH AMENDMENT RIGHT TO COUNSEL RIGHT TO COUNSEL

Based on right not be be Based on 6th Amendment right to compelled to incriminate counsel

Attaches when custodial Attaches when formal adversarialinterrogation proceedings initiated

Not applicable to undercover If deliberate elicitation, even byactivities (no compulsion) undercover agent of govt, applicable

Even though can have functional Courts more likely to find deliberateequivalent of interrogation, narrowly elicitation (when no actual questions)construed & focuses on suspect Focuses on state of mind of officer

Custody (not offense) specific Offense (charge) specific

Not constitutionally based so, in FOPT applies to violations (onegeneral, FOPT does not apply narrow exception but we did not cover)to violations (exception–Seibert)

BOTH REQUIRE KIV WAIVER; VALID WAIVER OF MIRANDA RIGHT TO COUNSEL SUFFICIENT FOR WAIVER OF 6TH RIGHT TO COUNSEL DURINGPRE-TRIAL INTERROGATION

STATEMENTS THAT VIOLATE EITHER 5TH OR 6TH RIGHT TO COUNSEL CANBE USED TO IMPEACH DEFENDANT AT TRIAL

92

Page 93: Crim Pro Outline

1 EFFECTIVE ASSISTANCE OF COUNSEL

6TH AMENDMENT–

Application to states through due process clause of 14th

1963—Gideon v. Wainwright—right to counsel (appointed if too poor) in all felonies

1972—Argersinger v. Hamlin—right to counsel in all misdemeanors in which Defendant receives sentence of incarceration

Most states---right to counsel if offense involves POSSIBILITY ofincarceration

APPLIES TO EVERY CRITICAL STAGE

Post-indictment interrogation, lineups and showupsArraignmentsPreliminary hearingsPlea hearingsTrialsSentencing hearingsProbation revocation hearings1st Appeal of right

FARETTA V. CALIFORNIA (p. 973—not assigned)

Defendant has a constitutional right to proceed without counsel when he voluntarily andintelligently elects to do so.

WHAT LEVEL OF COMPETENCE MUST LAWYER EXHIBIT TO SATISFY 6TH???

Strickland v. Washington (p. 989)

Does 6th apply to capital sentencing procedure???

Is it enough that counsel present alongside D at trial???

What is enough????

2 prong test

93

Page 94: Crim Pro Outline

1. Defendant must show that counsel’s performance deficient

2. Defendant must show that deficient performance prejudiced D (deprived D of fair trial)

A BUT FOR TEST—reasonable probability that but for counsel’s deficient performance,

result would have been different

HOW JUDGE WHETHER DEFICIENT???

1. Presume reasonable2. Did counsel perform basic duties?3. Were prevailing norms of practice and ethics followed?4. Given all circumstances, were actions reasonable??5. Not judging on hindsight.

APPLICATION TO FACTS IN STRICKLAND

Facts????

What did counsel do??

What did he not do???

Reasons for not doing??

Application of standards to these facts-----Was counsel deficient???

Even so----would result have been different???

CONTRAST

—Wiggins v. Smith—p. 1013

Failure to investigate mitigating evidence WAS deficient and WAS prejudicial

—Rompilla v. Beard—p. 1013

Failure to investigate details surrounding prior conviction that state was using in argument to support death penalty WAS deficient and WAS prejudicial

COURT SEEMS MORE WILLING RECENTLY (THAN IN 1984) TO FIND DEFICIENT PERFORMANCE IN CAPITAL CASES

OTHER INEFFECTIVE

94

Page 95: Crim Pro Outline

Dretke v. Draughon, 427 F.3d 286 (5th Cir.2006)

Failure to obtain forensic examination of path of bullet that killed victim, given that distance from Defendant to Victim was critical to Defendant’s claim that he lacked intent to kill and there is reasonable probability that but for absence of such forensic evidence, result of trial would have been different.

Cathel v. Marshall, 428 F.3d 452 (3rd Cir. 2006)

Counsel completely failed to prepare for penalty phase of capital case—failed to interview more than 10 witnesses who would have testified on D’s behalf; failed to gather any documentary evidence; failed to engage experts on mitigation material; failed to request continuance for penalty phase and instead agreed with prosecutor within 1 hour after guilty verdict to stipulate to single aggravating and mitigating factor; bland, emotionless argument that made no plea for Defendant’s life.

Poole v. Henry, 409 F.3d 48 (2nd Cir. 2006)Attorney promised in opening statement to present alibi defense but elicited alibi for wrong date and continued to press fallacious alibi on jury even after it was clear that alibi was for night after crime.

PROBLEM—NOTE 10A

Ineffective assistance??

PROBLEM NOTE 10B

Ineffective assistance??

NOTE 12—P. 1015

Illustrates that it is very difficult to prevail when issue raised.

However, it is frequently raised on appeal—sometimes it’s the only claim a defendant has.

95

Page 96: Crim Pro Outline

96

Page 97: Crim Pro Outline

97